Download A 57-year-old woman with coronary artery disease associated with

Survey
yes no Was this document useful for you?
   Thank you for your participation!

* Your assessment is very important for improving the workof artificial intelligence, which forms the content of this project

Document related concepts

Prenatal testing wikipedia , lookup

Dental emergency wikipedia , lookup

Medical ethics wikipedia , lookup

Patient safety wikipedia , lookup

Dysprosody wikipedia , lookup

Adherence (medicine) wikipedia , lookup

Electronic prescribing wikipedia , lookup

Patient advocacy wikipedia , lookup

List of medical mnemonics wikipedia , lookup

Transcript
A 57-year-old woman with coronary artery disease associated with hyperlipidemia
comes to the clinic for a scheduled follow-up appointment. She saw you for the first time
approximately 1 month ago to establish care. In the interim, she was started on
hydrochlorothiazide for elevated blood pressure (confirmed on a repeat nurses visit) and
on simvastatin for a fasting LDL of 190. She has a remote history of alcoholism, but
denies any alcohol intake over the past 10 years. Today, she complains of mild,
generalized weakness and states that her shoulders and thighs are "achy". She denies
rhinorrhea, fevers, chills, nausea, vomiting, or diarrhea. While she does not complain of
any dysuria, she states that her urine has been very dark for the past few days. She
denies abdominal or flank pain. Laboratory studies show a mildly elevated white blood
cell count, a normal hematocrit, and normal electrolytes. Her AST (or SGOT) is 415 and
her ALT is 25. Bilirubin and alkaline phosphatase are within normal limits. The most
appropriate next step in evaluation is to
A. determine creatinine kinase level
B. obtain an erythrocyte sedimentation rate
C. order a GGT level and a serum alcohol level
D. send Hepatitis A, B, and C serologies
E. send her for a right upper quadrant ultrasound
Explanation:
The correct answer is A. AST is less specific for liver than ALT. AST is found in multiple
organs and will be elevated with any muscle injury. In fact, before the advent of assays
for the MB fraction or troponin, AST was used to assess for myocardial infarction. The
patient was recently placed on simvastatin (an HMG Co-A reductase inhibitor). While
these cholesterol-lowering drugs are generally benign, myositis is a complication that
the prescribing physician must be aware of. The patient's presentation of fatigue and
muscle aches fits the diagnosis of myositis. In addition, an elevated AST with an
otherwise completely normal liver panel should heighten suspicion that the AST is not
coming from the liver. The patient's dark urine is classic for myoglobinuria. The patient
should be treated with intravenous fluids to maintain renal perfusion, therefore avoiding
renal tubular injury from the myoglobin. Depending on the level of the creatinine kinase,
alkalinizing the urine may also help protect the kidney from injury in this setting. There
is no definitive treatment for the myositis itself. Typically, the myositis resolves after the
offending agent (simvastatin in this case) is discontinued.
An erythrocyte sedimentation rate (choice B) is incorrect. While an erythrocyte
sedimentation rate may be useful as a sensitive marker of inflammation, it is not specific
for any disease process. As this case illustrates, recently prescribed medicines should
always be considered at the top of your differential diagnosis as the etiology of a new
disease process. The erythrocyte sedimentation rate would not help make the
diagnosis, nor would the result change management.
An elevated AST to ALT ratio may be suggestive of alcoholic liver injury. This is thought
to be due to the fact that ethanol decreases ALT synthesis. In addition, ethanol is
thought to cause mitochondrial damage in the liver, where AST lives. However, the ratio
of AST to ALT in alcoholic hepatitis is more frequently closer to 2:1. The extremely high
ratio in this case (again, with a normal ALT) points to an extrahepatic process. An
elevated GGT can reflect alcoholic liver damage, but again, this is unlikely in the face of
1
completely normal bilirubin and alkaline phosphatase. Therefore, ordering a GGT level
and a serum alcohol level (choice C) is not correct.
Hepatitis A, B, and C serologies (choice D) are incorrect because the viral hepatitides
should not cause an isolated level in AST.
A right upper quadrant ultrasound (choice E) is incorrect mainly for the same reasons
as elucidated above. A right upper quadrant ultrasound is useful for evaluating
suspected structural disease. Typically, structural disease of the liver is suspected
when there are clues of hepatic obstruction. Elevated bilirubin and alkaline phosphatase
are typically elevated in hepatic obstruction, and both are normal in this case.
A 20-year-old man comes to your office with a dog bite to his left thigh received after
he and a friend taunted a neighbor's dog. He reports that the bite occurred about 36
hours ago and only came to your office after coworkers informed him that dog bites
frequently become infected. His temperature is 37.0 C (98.6 F), blood pressure is
110/70 mm Hg, pulse is 63/min, and respirations are 13/min. On physical examination
you notice a shallow abrasion on his left thigh, which is mildly tender. There is no
surrounding edema or erythema. The most appropriate management of this patient is
to
A. administer a rabies vaccination
B. prescribe amoxicillin-clavulanate
C. prescribe clindamycin
D. prescribe penicillin
E. provide local wound care without antibiotic therapy
Explanation:
The correct answer is E. Only 5-20% of dog bites become infected. This patient does
not seem to have any systemic signs of infection and his wound doesn't appear to be
infected. Puncture wounds become infected more often than abrasion and this patient
doesn't have any skin punctures. Local wound care including debridement, cleaning,
and irrigation are essential. X-rays can be taken if fracture is suspected but a fracture
would be unlikely in a superficial thigh wound.
Rabies vaccination (choice A) is not indicated now. Vaccination for rabies should be
considered only when you have a strong suspicion of rabies. Rabies is always fatal so
treatment is essential if the disease is seriously being considered (such as a bat,
skunk, raccoon, fox, bat, etc.). In this case, the dog was a household pet, which
probably indicates that it was vaccinated in the past. Also, the dog did not seem rabid
(the dog bite occurred only after being provoked). If the dog spontaneously bit the
patient, suspicion might be higher. If rabies is suspected, an attempt to contact the
dog owner is a reasonable first step to determine the pet's vaccination status. Local
wound care remains important.
Antibiotics would be indicated for patients with signs of local or systemic infection.
Hand bites should also be treated with antibiotics since infection can be devastating to
a patient. Amoxicillin-clavulanic acid (choice B) and clindamycin (choice C) would both
be acceptable antibiotics since they have broad spectrums covering likely infectious
organisms such as Pasteurella multocida, Staphylococcus, Streptococci, and
anaerobes.
Penicillin (choice D) is a well studied choice for prophylaxis of hand bites but concerns
2
about narrow spectrum of activity have caused many physicians to use alternative
antibiotics.
A 32-year-old man is admitted to the hospital following a motor vehicle accident. He was
driving a car at 70 miles per hour and was involved in a head-on collision. He was
wearing a seat belt at the time of the accident. Initial evaluation in the emergency
department by means of physical examination, cervical spine, and chest and pelvic xrays did not reveal any abnormalities. A complete blood count and metabolic panel were
within normal limits. Urine analysis does not show evidence of blood or RBCs. He is
admitted to the hospital because of suspected contusion to the brain and because of the
death of the passenger sitting in the front seat in the same accident. Twelve hours after
admission, he complains of severe abdominal pain. His vital signs are within normal
limits except for tachycardia. His abdomen is soft, slightly distended with absent bowel
sounds. Repeat laboratory investigation shows a stable hematocrit, leukocyte count of
16,000/mm3, and a serum amylase of 4000 U/L. The most appropriate next step in the
management of this patient is
A. analgesics
B. CT scan of the abdomen and pelvis with IV contrast
C. diagnostic peritoneal lavage
D. observation
E. ultrasonography
Explanation:
The correct answer is B. Pancreatic injuries should be suspected in blunt abdominal
trauma. In motor vehicle accidents associated with seat belt injuries, pancreatic
fractures is a known entity. These patients initially may not have any symptoms and
signs and may not reveal any abnormality in laboratory investigations. Continued
observations sometimes reveal abdominal pain or a mildly tender distended abdomen.
Elevation of serum amylase should arouse the suspicion of injury to the pancreas. This
pancreatic fracture and disruption of the duct can be classically diagnosed with a CT
scan of the abdomen with intravenous contrast.
Analgesics (choice A) are not indicated without an established diagnosis in blunt
abdominal trauma.
Diagnostic peritoneal lavage (choice C) is not the right option for this patient, as it may
not reveal pathology in the retro peritoneum (i.e., pancreas). Diagnostic peritoneal
lavage is usually used in the initial investigation of the blunt abdominal trauma to rule
out any intraperitoneal bleeding or evidence of bowel injury.
Continued observation (choice D) is not the right option in the presence of symptoms
and signs in a patient with blunt abdominal trauma.
Ultrasonography (choice E) is not an optimum investigation method in the diagnosis of
a pancreatic fracture. It is technician-dependent and subjective. Ultrasonography in
blunt abdominal trauma is useful in the initial investigation to rule out any fluid
collection, i.e. blood.
A 21-year-old man is brought by ambulance to the emergency department after being
3
involved in a high-speed collision on his motorcycle. The patient struck a retaining wall at
over 100 miles per hour. At the scene, he was non-responsive. He was intubated, a
peripheral intravenous was placed, and he was transported to the hospital. On arrival,
the patient is non-responsive to command. His temperature is 37.0 C (98.6 F), blood
pressure is 60/30 mm Hg, pulse is 140/min, and respirations are 10/min via mechanical
ventilator. He has multiple ecchymoses on his abdomen and chest, with an open femur
fracture on the right and a depressed skull fracture. His abdomen is distended and tense
and a radiograph suggests massive blood in the abdomen from a venous tear. A femoral
vein cut-down is performed by the surgical team and a femoral vein central line is
placed. After rapid infusion of 6 liters of crystalloid and 4 units of packed red cells via the
femoral line, the patient is noted to be in pulseless electrical activity. The most
appropriate next step in the management of this patient is to
A. bring the patient emergently to the operating room
B. continue rapid transfusion of blood products and crystalloid via the femoral line
C. give epinephrine, intravenously
D. place an upper extremity intravenous line and infuse volume and blood
product
E. perform DC cardioversion at 200 Joules
Explanation:
The correct answer is D. The utility of central lines are related to the nature of the injury
or clinical condition of the patient. For example, internal jugular lines are useful when
the shoulders or chest is going to be operated on. For this patient who likely has an
inferior vena cava rupture, a precaval line does nothing except deliver blood product to
his vena cava that promptly discharges it through a defect into the abdominal space.
This patient is now in PEA because he has continued to lose volume through his defect.
An upper extremity line should be placed (utilizes SVC) and volume should be delivered
in that manner.
Attempting to bring the patient emergently to the operating room (choice A) could result
in his death. Patients must be stabilized (the ABCs of resuscitation) before any
additional interventions are taken. This patient is in an ACLS arrest and must be
resuscitated prior to being discharged from the emergency department to the operating
room.
For the reasons discussed above, to continue rapid transfusion of blood products and
crystalloid via the femoral line (choice B) would be ineffective at restoring this patient's
pressure.
Similarly, giving epinephrine intravenously (choice C) is not correct because this is only
called for in pulseless VT or VF.
DC cardioversion at 200 Joules (choice E) is incorrect because this patient is in PEA
arrest. This ACLS algorithm calls for interventions that restore the circulatory tone.
There is no evidence that this patient is in pulseless ventricular tachycardia or
fibrillation, which does call for countershock.
A 71-year-old man is undergoing a coronary artery bypass procedure in the
morning. He has a long-standing history of coronary artery disease and
hypertension and takes atenolol, furosemide, captopril, and digoxin daily. The
patient also suffers from moderate osteoarthritis of the cervical spine. You are
4
called to insert the internal jugular central line that will be used during the
case the following morning. It is difficult to get the patient in optimal position
but the attempt is made at placing the line. After multiple attempts on the
right, a successful left-sided internal jugular central venous catheter is placed.
The position is confirmed by chest radiograph. Three hours later, the nurse
calls because the patient is now stridorous. On arrival to the room, you notice
that the patient is having difficulty breathing, has audible inspiratory stridor,
and has a massively swollen right neck extending to the midline and to the
clavicle inferiorly. The next step in the management of this patient is to
A. administer a bolus of furosemide, intravenously
B. apply pressure to the right side of the neck
C. call for a vascular surgeon emergently
D. give nebulized beta agonist therapy immediately
E. perform a bedside surgical opening of the neck
Explanation:
The correct answer is C. All central line placements are associated with
complications. The nature and incidence of those complications are site
specific. For internal jugular lines, the incidence of carotid puncture is about
5%. Most of these are trivial since the puncture occurs with the small 19G or
22G seeker needle, but in patients where the anatomy is difficult, multiple
attempts at line placement often mean multiple lacerations of the carotid.
This patient has bled into his neck and has nearly compressed his trachea
from the pressure of the contained blood (the blood is under pressure
equivalent to his mean arterial pressure). A surgeon should be called
immediately for either an emergency bedside opening and decompression,
or to go to the operating room emergently.
Since there is no evidence that this patient is in pulmonary edema, a bolus
of intravenous furosemide (choice A) will not be helpful.
Trying to apply pressure to the right side of the neck (choice B) is incorrect
and could result in the collapse of the trachea. The bleeding has already
stopped secondary to the tamponade effect of the closed space, any further
extrinsic compression will only further compromise his airway.
Similarly, giving nebulized beta agonist therapy (choice D) is not useful since
the stridor heard is not due to bronchiolar spasm but to external
compression of the extra-thoracic trachea.
Performing a bedside surgical opening of the neck (choice E) should ONLY
be attempted by an experienced vascular surgeon. This is because once the
tamponade is relieved, the carotid will continue to bleed and unless
hemostasis can be attained, the patient runs a severe risk of bleeding to
death.
A 49-year-old man comes to clinic for follow up and monitoring of his oral
anticoagulation levels. The patient is postoperative day 62 from a left total
knee replacement. On postoperative day number 2 he suffered a pulmonary
embolism. He was placed on intravenous unfractionated heparin and then
oral warfarin. He was discharged home with follow-up instructions to return to
5
the clinic for monitoring of his prothrombin time/international normalized ratio
(INR) every 3 weeks. On return to the clinic today his PT/INR is found to be
22.4/7.3. His physical examination is unremarkable. The most appropriate
management at this time is to
A. admit the patient to the hospital
B. instruct the patient to discontinue warfarin and return in 1 week
C. instruct the patient to discontinue warfarin week until his next visit
in 3 weeks
D. give protamine sulfate, intravenously
E. give vitamin K and follow up with the patient at his next visit
Explanation:
The correct answer is A. This patient has a supratherapeutic INR and is at
great risk of spontaneous bleeding. He requires hospitalization, vitamin K
administration, and if there is bleeding, fresh frozen plasma. Because he is
within 2 months of a pulmonary embolism, his risk of recurrent PE off of
warfarin is high and he will therefore need to be restarted on heparin as
soon as his INR falls below 2.0. This management can only be
accomplished in the hospital.
Although withholding warfarin is appropriate, not admitting the patient to the
hospital (choice B) and (choice C) is incorrect for the reasons stated above.
Protamine sulfate (choice D) is an antagonist of unfractionated heparin. It
has no effect on warfarin, which interferes with the carboxylation of factors 2,
7, 9, and 10.
Again, administration of vitamin K (choice E) is appropriate in this setting,
but because the patient's risk of bleeding is so high, he should be admitted
to the hospital for the management.
A 57-year-old woman with diabetes and hypertension comes to the office because of a
5-day history of weakness and lethargy. The symptoms developed slowly but have
worsened over the last 2 days. She denies any chest pain, dyspnea, fever, cough,
abdominal pain, or dysuria, but has noticed a decrease in her urine output. One week
prior to admission she underwent a CT scan of the abdomen with intravenous contrast
for routine follow up of an abdominal aortic aneurysm, which is stable at 4 cm. She
regularly takes hydrochlorothiazide, glyburide, captopril, and aspirin. Her temperature is
37.0 C (98.6 F), blood pressure is 165/94 mm Hg, pulse is 92/min, and respirations are
14/min. Physical examination shows 2+ pitting edema in the lower extremities and 1+
peripheral pulses. Urinalysis shows trace protein, 1-3 white blood cells, tubular casts and
no red casts. Laboratory studies show:
6
This condition most likely would have been prevented by
A. administering acetylcysteine prior to the contrast for the CT scan
B. holding her captopril 2 days prior to the CT scan
C. giving her intravenous ampicillin during the CT scan
D. giving her intravenous furosemide 1 hour prior to the CT scan
E. premedicating her with nifedipine 2 days prior to the CT scan
Explanation:
The correct answer is A. The patient is suffering from acute tubular necrosis (ATN)
presumably secondary to the contrast she received during her CT scan. Contrast
induced ATN is more common in patients with diabetes and baseline renal insufficiency.
The urinalysis is also suggestive of this diagnosis with the presence of tubular casts
and the absence of red cell casts. Recent studies have shown that the incidence of
contrast nephropathy can be reduced with the use of acetylcysteine prior to the
contrast.
There is no evidence to suggest that holding captopril (choice B) will reduce the
incidence of contrast induced nephropathy. ACE inhibitors have been shown to slow the
progression of renal disease in diabetics with proteinuria.
Premedication with antibiotics (choice C) has no role in the prevention of contrast
nephropathy. They are indicated in the prophylaxis of bacterial endocarditis in patients
with valvular heart disease undergoing selected invasive procedures.
Diuretics (choice D) have not been shown to be beneficial in the prevention of contrast
nephropathy. Adequate hydration should be provided to patients at risk prior to studies
with contrast. Of course, if patients develop clinical evidence of volume overload
diuretics should be used judiciously.
Calcium blockers (choice E) have also not shown any benefit in the prevention of ATN
and should not be used for that purpose. If the patient is already taking them however,
there is no reason to discontinue them prior to a contrast study.
A 44-year-old man with metastatic liver cancer requires a central line for total parenteral
nutrition. The patient was otherwise healthy until 3 months ago at which time he was
diagnosed with liver cancer. A subsequent workup for metastatic disease disclosed that
the tumor had already spread to his lungs, abdominal viscera, and brain. He is
7
scheduled to begin chemotherapy and radiation therapy and will require nutritional
support. The patient is given informed consent and the details of the procedure are
discussed with him. A decision is made for a right subclavian line. The patient is
positioned, prepped, and draped in a sterile manner and the skin is anesthetized with
1% lidocaine. During the procedure, the guidewire slips from your fingers and
disappears through the lumen of the catheter. This patient is at greatest risk for
A. atrial-septal perforation
B. cardiac arrhythmia
C. pneumothorax
D. tricuspid valve damage
E. ventricular perforation
Explanation:
The correct answer is B. Wire loss during insertion of a central venous line is a
relatively common occurrence with an estimated incidence of about 1%. Once lost, the
issue becomes where the wire will become lodged. The most common location is in the
distal pulmonary artery such that the proximal tip of the wire is in the right ventricle.
When this occurs, it is not uncommon, in fact it is quite normal to see arrhythmia. The
problem is when these rhythms are ventricular tachycardias. The wire must be removed
by an interventional radiologist or cardiac surgeon.
Unless the wire lodges in the atrium and it is somehow "pushed" during extraction,
atrial-septal perforations (choice A) are uncommon.
A pneumothorax (choice C) is not a complication of wire loss, but of placing subclavian
central lines.
Again, unless some force is applied to the wire during attempted extraction, tissue
damage to the tricuspid valve (choice D) is not very common.
A ventricular perforation (choice E) could occur if the wire is coiled in the ventricle. The
force of ventricular contraction could force the tip of the wire through the membranous
septum. Although this is possible, the most common, in fact, routine complication from
wire loss is cardiac arrhythmia.
An 11-year-old boy is rushed into the emergency department following a motor vehicle
accident. The patient was a restrained, front seat passenger when an oncoming car hit
the passenger side of the car. The boy denies loss of consciousness, but in the hospital
he complains of pain over his right side. His pulse is 139/min, blood pressure is 118/59
mm Hg, and respirations are 24/min. On physical examination he has decreased breath
sounds over the right base and there is ecchymosis over the right flank. His abdomen is
soft with tenderness in the right upper quadrant. Appropriate management of his airway
with neck stabilization is provided and he is resuscitated appropriately. Imaging studies
of his neck are negative and a chest X-ray does not show a pneumothorax or rib
fracture. Dipstick of spontaneously voided urine is positive for blood. Urinalysis confirms
the presence of 50 RBCs/hpf. In regards to his hematuria the next most appropriate
course of action is to
A. begin empiric antibiotics
8
B. obtain CT scan of abdomen and pelvis
C. order renal/bladder ultrasound
D. perform retrograde urethrogram
E. place Foley catheter
F. repeat urinalysis
G. schedule outpatient intravenous pyelogram (IVP)
Explanation:
The correct answer is B. The management of hematuria associated with trauma differs
in adults and children. In the adult population, imaging is performed only in those
patients with gross hematuria or microscopic hematuria plus hypotension. This differs
from the pediatric patient. In children, any degree of hematuria (gross or microscopic)
should be investigated with imaging studies. One reason for this discrepancy is that
large amounts of catecholamines released in injured children may sustain blood
pressure in the face of hypovolemia. A CT scan is the most useful imaging modality in
this setting. A CT is noninvasive, accurate and fast, and it can help in assessing the
size and extent of retroperitoneal hematomas and renal parenchymal trauma. Not only
does this child have microscopic hematuria (an indication by itself to perform imaging
studies), but he also has signs (flank ecchymosis and tenderness) that raise the
suspicion of renal injury. High suspicion for renal injury (i.e., rib fracture, flank
contusion, deceleration injury) is another indication to perform imaging studies.
There is no indication at this time that the patient requires antibiotics (choice A). He has
no open fractures, large abrasions, or burns. Further diagnostic studies must be
undertaken to determine if the patient requires antibiotics for any disruption to the
urinary system.
Ordering a renal/bladder ultrasound (choice C) acknowledges the fact that the patient
does require investigation of his urinary system. However, a CT scan is quicker and will
have a much higher yield for associated urinary and abdominal injuries versus
ultrasound.
A retrograde urethrogram (choice D) allows for visualization of the urethra to investigate
for extravasation. It is performed when there is gross blood at the meatus. It will not
help in this patient whose injuries are suspected to be intraabdominal.
Hematuria is not an indication for Foley catheter placement (choice E). As long as the
patient is awake without altered sensorium he should be given an opportunity to void on
his own (which he has done). It should be noted that in any patient who has gross blood
at the meatus a retrograde urethrogram must be obtained prior to placing a Foley
catheter.
Repeating the urinalysis (choice F) will only delay this patient's work-up. Although the
urine dipstick may be falsely positive for heme, there is no reason to doubt the validity
of the urinalysis. Besides, this patient's flank ecchymosis and tenderness warrant
imaging independently of his urinalysis results.
Intravenous pyelogram, or IVP (choice G), has a role in evaluating the urinary tract.
However, with the use of CT scan, IVP has a very limited role in evaluating
renal/ureteral trauma. Delaying imaging until this patient is an outpatient could mean
missing a potentially life-threatening renal injury.
9
A 29-year-old truck driver is transferred to the general medical floor from the
intensive care unit where he was being treated after sustaining multiple
fractures from a motor vehicle accident. It is determined that the patient was
under the influence of alcohol at the time of the accident. He has been in the
intensive care unit for 20 days following open reduction and internal fixation
of bilateral femur, tibia, and fibula fractures. The patient is otherwise doing
well and has full use of his upper body. Legal proceedings will begin
subsequent to discharge, and the patient states he is committed to ceasing
alcohol consumption. Past medical history is significant for alcoholism.
Medications include imipenem and morphine administered via patientcontrolled analgesic pump. At this time, the patient is at greatest risk for
A. alcohol withdrawal
B. deep venous thrombosis
C. fat emboli to lungs
D. narcotic addiction from the morphine
E. suicide
Explanation:
The correct answer is B. This patient is profoundly immobile due to multiple
fractures of both legs and he is at increased risk for developing a thrombus
in the veins of his lower extremities. Deep venous thrombosis (DVT)
prophylaxis with subcutaneous heparin should already have been started.
DVT prophylaxis is necessary in all immobile patients, especially those who
have sustained major orthopedic injury.
Alcohol withdrawal (choice A) usually occurs a few hours to 10 days after
the last drink. Most typically, it occurs in the first 48 hours. Symptoms
include acute psychosis with hallucinations, delusions, disorientation,
agitation, and tremors. In this patient, alcohol withdrawal is very unlikely to
start now, 20 days after the injury.
A fat emboli (choice C) from bone fractures occurs in close temporal
proximity to the fracture or to the surgery. The patient is now 10 days
postoperative and 20 days after the initial injuries, making this unlikely.
Narcotic addiction (choice D) from patient controlled analgesia is very rare
and allows the patient to participate in his medical care. Care must always
be taken to balance analgesia and addiction potential.
Suicide (choice E) is unlikely given the hopeful outlook of this patient and
lack of history of depression or suicidal tendencies.
A 70-year-old man with hypertension, diabetes mellitus, and hypercholesterolemia is
postoperative hour number 8 after a laparoscopic cholecystectomy. He was diagnosed
with cholecystitis 2 days earlier and underwent an uneventful procedure. Now the patient
is on the floor for postoperative observation. His medical history is remarkable for severe
ischemic cardiomyopathy and he is status-post 2 myocardial infarctions. He has known
congestive heart failure with an ejection fraction of 70%. He also has severe aortic
stenosis with a valve area of less than 0.5 cm2. He takes maximal doses of a betablocker and calcium channel blocker daily, as well as enalapril and furosemide.
10
Preoperatively his temperature was 38.0 C (99.6 F), blood pressure was 180/85 mm Hg,
pulse was 44/min, and respirations were 18/min. Intraoperatively his pulse was stable at
40/min and his blood pressure was 150/70 throughout the case. On arrival to the floor,
you give him 0.1mg of atropine sulfate intravenously to treat the bradycardia. His pulse
rises to 140/min with a blood pressure of 120/40 mm Hg. He begins to complain of
severe shortness of breath and becomes mildly hypoxic on room air. The most
appropriate therapy at this time is to
A. administer edrophonium, intravenously
B. administer furosemide, intravenously
C. administer propranolol, intravenously
D. obtain a chest radiograph and a 12 lead electrocardiogram
E. perform endotracheal intubation
Explanation:
The correct answer is C. This patient has severe diastolic dysfunction secondary to his
hypertension, severe aortic stenosis, and ischemic heart disease. This can be
appreciated by the fact that he has known congestive heart failure and has an ejection
fraction of 70%, greater than normal for his age. This represents a smaller end-diastolic
volume with preserved stroke volume. These patients are prone to dramatic increases
in pulmonary pressures with tachycardia of any cause. They are therefore prone to
pulmonary edema and congestive failure episodes with tachycardia. This is the reason
why he is so heavily beta blocked. There is no indication to treat a sinus bradycardia
that is without symptoms. The patient must now have his heart rate slowed with betablockers (propranolol).
To antagonize the atropine, intravenous edrophonium (choice A) should not be
considered. Such a combination is used for reversal of neuromuscular blockade during
anesthesia, but can have severe effects such as a dramatic bradycardia and even
asystole in patients who are heavily beta blocked and on nodal agents such as calcium
channel blockers, as in this patient.
Intravenous furosemide (choice B) is appropriate management for the pulmonary
edema once the precipitating cause of the edema is reversed. Diuretics lower venous
filling pressures and therefore decrease pulmonary congestion.
Obtaining a chest radiograph and a 12 lead electrocardiogram (choice D) does not take
precedence over the acute management of the patient's symptoms. If there was
uncertainty as to the cause of the symptoms, these tests may be able to clarify the
cause and therefore direct therapy.
Although this patient is in mild respiratory distress from the pulmonary edema, there is
no evidence of severe hypoxia, hypercarbia, obtundation, or altered mental status that
dictates the need to perform tracheal intubation (choice E).
A 64-year-old farmer comes to the clinic with an injury to the foot, which happened at his
farm 24 hours ago. He reports that he was working on the farm, when he accidentally
stepped on a rusty nail, which penetrated deep into his foot. He took some analgesics
and he was feeling well. His wife urged him to come to the clinic. He denies any fever,
chills, or rigor. His past medical history is significant for non-insulin dependent diabetes
mellitus, which is well controlled with oral therapy. He has no past surgical history.
11
Examination of the foot reveals a deep penetrating wound in the sole of the left foot.
There is no associated erythema or induration around the foot. No foreign particles are
noted along the edges of the foot, which is tender on palpation. There is no motor or
sensory loss. No active bleeding is noted in the penetrating wound. The patient tells you
that he had 3 doses of tetanus toxoid injections when he was young. The last tetanus
injection was 8 years ago for a similar episode. The most appropriate next step in
management is to
A. administer a tetanus toxoid booster injection
B. administer a tetanus toxoid booster injection and human immunoglobulin
C. provide human immunoglobulin
D. provide no additional therapy at this time
E. surgically debride the wound
Explanation:
The correct answer is A. Tetanus prone wounds are any wounds that are over 6 hours
old, deep, crushed or penetrated, contaminated with soil, associated with compound
fractures, partial or full thickness burns, or human or animal bites. Patients who
received 3 doses of tetanus toxoid in the last 5 years do not need further therapy for a
tetanus prone wound or for a clean wound. Patients who received a tetanus toxoid
between 5-10 years ago and have a tetanus prone wound need booster tetanus toxoid
injections, whereas those with a clean wound would need no further therapy. When the
last dose of tetanus toxoid was more than 10 years ago, then both clean wounds and
tetanus prone wounds need tetanus toxoid booster injections. In addition, those with a
tetanus prone wound, whose last dose was more than 10 years ago, need to have
human immunoglobulin (choice C) administered.
When the tetanus toxoid vaccination history is not available or the tetanus toxoid
injections were administered more than 10 years ago, then an individual with a tetanus
prone wound would need tetanus toxoid and human immunoglobulin (choice B).
Whereas in a clean wound, a tetanus toxoid complete course is essential. In this case,
the farmer received a tetanus toxoid booster 8 years ago and the wound described is a
tetanus prone wound. Hence, one booster dose of tetanus toxoid is indicated in this
patient.
No therapy (choice D) is not the correct option in this patient, as the wound is tetanus
prone, and the last toxoid booster was given more than 5 years ago.
Surgical debridement is indicated in crushed wounds. Wherever there is a soft tissue
injury from crushed wounds, and when there is some devitalized tissue, surgical
debridement is indicated, even if it is old. In a clean, penetrating wound, surgical
debridement (choice E) is not indicated.
A 19-year-old boy comes to the office because of a cough for the past 2 weeks. It was
initially a "dry, annoying cough," but it has recently changed to a "wet cough with
yellowish-green sputum." He is a college freshman and is "pledging" a fraternity. He
admits to many nights of "hard drinking." You notice that his front tooth is missing and he
says that he lost it 2 weeks ago during a "hazing" event when the pledges were forced to
"funnel" beer. Her temperature is 37.8 C (100.0 F), blood pressure is 110/80 mm Hg,
and respirations are 20/min. Physical examination shows wheezes over the right lung
12
base. It is otherwise unremarkable. The most appropriate next step is to
A. admit him to the hospital for immediate bronchoscopy
B. order a chest x-ray
C. order pulmonary function tests
D. order tomography of the right lower lobe
E. prescribe antibiotics and send him home
Explanation:
The correct answer is B. This patient most likely has a foreign body aspiration (his
tooth). A foreign body aspiration typically presents with a new cough, wheezing and the
patient often has a history of heavy drinking, drug overdose, or trauma. This patient was
"funneling" beer, which is when a large funnel and a tube are used to supply a large
amount of beer into the recipient's mouth at a rapid speed. A chest x-ray is the initial
study and may show a radiopaque object and postobstructive pneumonia or abscess.
Management is by bronchoscopy combined with antibiotics.
Admitting him to the hospital for immediate bronchoscopy (choice A) is inappropriate
because while it seems likely that he has a foreign body aspiration (his tooth), a chest
x-ray should be ordered first. He should be evaluated for a radiopaque object and
postobstructive pneumonia, abscess or another process that may be causing these
symptoms.
Pulmonary function tests (choice C) are inappropriate at this time. This patient most
likely has a foreign body aspiration, not asthma. The wheezing that is heard over the
right lower lobe is most likely caused by obstruction of a bronchus.
Tomography of the right lower lobe (choice D) is not the next step in evaluating this
patient who most likely has a foreign body aspiration. A chest x-ray should be ordered.
Since this patient most likely has a foreign body aspiration, it is inappropriate to
prescribe antibiotics and send him home (choice E). He requires removal of the object
and antibiotics. Sending him home with antibiotics may be appropriate if he is a healthy
patient with a cough and a suspected mycoplasma pneumonia infection, however a
chest x-ray is usually indicated in this situation too.
A 45-year-old man is struck by a motor vehicle and is transported by a Med-Flight to the
local emergency department. The patient is reported to be intoxicated with a Glasgow
score of 15 on the scene of the accident. He was struck by a vehicle while crossing the
street. On arrival, the patient is awake and somewhat combative. He is alert to person
only. Primary survey reveals a well-developed man in mild distress. He is in a cervical
collar. His blood pressure is 150/90 mm Hg, his heart rate is 130 /min, and he is
breathing at 26/min. He has obvious lower extremity tibia fractures bilaterally and a
laceration on his forehead. Secondary survey reveals severe pelvic trauma with bilateral
inferior and superior pubic ramus fractures. The most appropriate diagnostic test at this
time is
A. abdominal CT scan
B. abdominal ultrasound
13
C. diagnostic peritoneal lavage
D. exploratory laparotomy
E. pelvic ultrasound
Explanation:
The correct answer is A. This patient has severe trauma involving his lower extremities
and pelvis. The concern is for intraabdominal trauma resulting in bleeding. An
abdominal CT scan is the imaging modality of choice since it detects both
intraabdominal and retroperitoneal blood.
An abdominal ultrasound (choice B) has been shown to be equivalent to diagnostic
peritoneal lavage (choice C) for detection of free intraabdominal blood. However,
neither of these modalities can reliably diagnose the presence of a retroperitoneal
bleed.
Although some centers immediately opt for an exploratory laparotomy (choice D), this is
actually not the appropriate diagnostic step. This patient will certainly go to the
operating room for repair of his tibial fractures and stabilization of his pelvis. The
addition of a midline abdominal incision to his procedure list would entail a tremendous
amount of OR time and additional chance of morbidity for the patient. The most
appropriate intervention is to look for the presence of blood and if the clinical picture still
suggests a bleed, but there has been none disclosed by imaging, then a surgical
exploration is warranted.
A pelvic ultrasound (choice E) offers no diagnostic benefit for the evaluation of an
intraabdominal bleed.
A 3-year-old recently adopted boy is brought to the office for the first time by his new
parents for a "check-up." They say that he seems to be adjusting to his new family
"pretty well", but that he is a bit "slower" than their other children and he has poor
socialization skills. He is able to walk unassisted. However, he cannot walk up steps
(even though his previous home had steps), and he is only able to say a few words.
They are also concerned because he looks a little different from their other kids, who
have started to make mean comments to him. Physical examination shows
microcephaly, epicanthal eye folds, small teeth, poorly formed concha, and a
holosystolic murmur. This condition could most likely been prevented by
A. dietary supplementation with vitamin D when he was an infant
B. maternal avoidance of alcohol during pregnancy
C. maternal avoidance of cocaine during pregnancy
D. maternal intake of folate before and during pregnancy
E. regular consumption of iron-containing formula when he was an infant
Explanation:
The correct answer is B. This patient most likely has fetal alcohol syndrome (FAS),
which could have been prevented by maternal avoidance of alcohol during pregnancy.
The classical features of this syndrome are growth retardation, abnormal facial features,
and central nervous system problems. Patients affected with FAS can have serious
14
lifelong disabilities, including mental retardation, behavioral problems, and learning
disabilities. It is unknown exactly how alcohol causes these specific abnormalities.
However, it is important to stress to all patients that no amount of alcohol should be
consumed during pregnancy. A nurturing and stable environment is especially important
for children with FAS.
Dietary supplementation with vitamin D when he was an infant (choice A) is incorrect
because vitamin D deficiency is associated with rickets, not FAS. Rickets is
characterized by a delayed closure of the fontanelles, craniotabes, bowing of the lower
extremities, and fractures.
Maternal avoidance of cocaine during pregnancy (choice C) is incorrect because
maternal cocaine usage is associated with hyperirritability, cerebral infarction, and
intrauterine growth retardation. It is not associated with the craniofacial abnormalities in
this case.
Maternal intake of folate before and during pregnancy (choice D) is important to reduce
the risk of neural tube defects (NTD). It is recommended that women of reproductive
age take folate daily. The patient in this case does not have a NTD.
Regular consumption of iron-containing formula when he was an infant (choice E)
would have prevented iron-deficiency anemia, which is associated with fatigue,
anorexia, irritability, pale skin and mucous membranes, and a decrease in cognitive
function. Mouth soreness and "spooning" of the nails may be present with severe
anemia. Iron-deficiency anemia is not associated with abnormal facial features.
A 30-year-old man who you have not seen in a few years comes to the clinic because of
a recent psoriasis flare up. You are talking about possible new stressors in his life that
might have precipitated this worsening. The patient agrees and tells you that he has
been traveling a lot recently. He explains that an "odd" thing happened when he went to
visit his ex-girlfriend who lives in another state. He ended up being admitted to the
psychiatric hospital because he was acting as if he were on "speed". He was diagnosed
with bipolar disorder and treated for several days. He was instructed to follow up with a
mental health center in his community and given an appointment for the next week.
Currently, he feels upset because he was trying to get back to his routine and feeling
confident. As his doctor, you are aware that you will need to encourage his compliance,
because the worsening of psoriasis could be ascribed to the medication he was placed
on. The medication that is most likely causing his skin condition is
A. carbamazepine
B. haloperidol
C. lithium carbonate
D. olanzapine
E. valproic acid
Explanation:
The correct answer is C. Side effects that occur with lithium carbonate are often
troublesome. These include sedation, cognitive difficulties, dry mouth, hand tremor,
increased appetite, polydipsia, polyuria, nausea, diarrhea, psoriasis, and acne.
Carbamazepine (choice A) can cause aplastic anemia and agranulocytosis as the most
15
serious side effects. The most common side effects include dizziness, ataxia, sedation,
dysarthria, nausea, hyponatremia, and cardiovascular conduction problems. Rarely can
it cause rash and exfoliation.
Haloperidol (choice B) related side effects generally are a consequence of the blockade
of dopaminergic D2 receptors. Apart from a spectrum of extrapyramidal symptoms, side
effects include sedation, weight gain, hyperprolactinemia, and neuroleptic malignant
syndrome.
Olanzapine (choice D) is a typical antipsychotic indicated in the management of
psychotic disorders and mood disorders because of its mood stabilizing properties. It
causes significant sedation, weight gain, postural hypotension, dizziness, dry mouth,
constipation, and other anticholinergic side effects. It rarely causes vesiculobullous
rash.
Valproic acid (choice E) most commonly causes short-term nausea, vomiting, diarrhea,
sedation, dizziness, and tremor. Alopecia and weight gain are long-term side effects.
A 9-year-old girl is brought to the office by her mother because it always
seems as if she is going to "burst from all of her energy." Her teacher says
that she has difficulty following instructions and waiting to be "called on" when
a question is posed to the entire class. This child has been disrupting the
class for "months" with her impulsive speech and "constant motion." "The
final straw" was when she knocked over 3 buckets filled with colored sparkles
when she was supposed to be sitting quietly at her desk. The mother is
concerned that she is going to be seriously injured "one of these days"
because of her distractibility and carelessness. It is almost "impossible" to get
her to go to bed at night. As the mother goes over her history, the child is
going through the drawers in the examining room. You explain that she
probably has attention-deficit hyperactivity disorder and you prescribe a high
dose of methylphenidate. You educate the patient and her mother about the
most common side effects of the medication, and tell them to call you if any of
them occur. Two weeks later, you receive a message that the mother called
the office because she believes that her daughter is experiencing a "very
frustrating side effect" of the medication. The most likely side effect that she
is referring to is
A. constipation
B. dystonia
C. insomnia
D. sedation
E. weight gain
Explanation:
The correct answer is C. Methylphenidate is one of the most common drugs
prescribed for attention-deficit hyperactivity disorder (ADHD). It is a
psychostimulant that produces positive behavioral responses in the majority
of individuals. Common side effects during the initial therapy include
insomnia, loss of appetite, abdominal pain, and weight loss. Growth
suppression with weight gain and/or inhibited body growth has been
16
reported with the long-term use of methylphenidate in children.
Constipation (choice A) is a common side effect of high potency
antipsychotic agents and other drugs with anticholinergic effects. It is not
typically seen with stimulants, such as methylphenidate.
Dystonia (choice B) is a side effect of high potency antipsychotic agents. It is
not a common side effect of methylphenidate.
Insomnia, not sedation (choice D), is a common side effect of
methylphenidate.
Weight loss is typically reported during treatment with methylphenidate, not
weight gain (choice E).
A 71-year-old man is postoperative day number 12 from a heart transplant. The patient
has a long-standing history of ischemic cardiomyopathy and successfully underwent a 5hour transplant from a 22-year-old donor. During the procedure he was started on his
immunosuppressive therapy that has continued. In the immediate postoperative period
he did well. He was extubated on day number 1, had his pulmonary artery and radial
arterial catheters removed on day number 3, and was transferred from the coronary care
unit on day number 4 with excellent pain control. Over the past 3 days however, he has
had increasing fever, lethargy, and mediastinal tenderness. The sternal wound appears
mildly erythematous but nonsuppurative. On palpation, the incision site is tender. The
most appropriate therapy is at this time is
A. broad-spectrum antibiotics
B. decreasing dose of immunosuppressive drugs
C. initiate antifungal therapy
D. surgical debridement
E. there is no therapy indicated
Explanation:
The correct answer is A. This patient likely has mediastinitis, a common postoperative
complication after heart transplant. The combination of the nature of the wound (median
sternotomy) with the profound immunosuppression creates a furtive environment for
mediastinal infection. Such patients require broad-spectrum antibiotic therapy.
Because of the transplant and the possibility of acute-rejection, decreasing the dose of
immunosuppressive drugs (choice B) is not tenable.
Bacteria cause most mediastinal infections in the immediate and intermediate
postoperative periods. Therefore, fungal infections in transplant recipients and the need
for antifungal therapy (choice C) are not usually an issue until 6-9 months of
immunosuppressive therapy.
Surgical debridement (choice D) is usually required when the sternum undergoes
dehiscence or there is necrotic tissue present that can become a culture medium for
additional infections. There is no indication that this patient's infection is that severe.
Because mediastinitis is a serious infection and can cause sternal dehiscence and
chronic infection, antibiotic therapy is indicated despite concerns for selecting for
resistant organisms (choice E).
17
You are evaluating a 34-year-old man in the emergency department following
an approximately 8 foot fall from a scaffolding while at work . He lost
consciousness at the scene, but regained consciousness and was
hemodynamically stable on transfer, as well as, upon arrival in the hospital.
On arrival, he was awake, but complaining of left-sided chest pain and leftsided facial pain. Of note, the patient has a history of alcohol and substance
abuse, and reports that he drank 3 beers this morning prior to work. His
physical examination is remarkable for normal mental status, some left
zygomatic abrasions, as well as abrasions below the left eye. Head, eyes,
ears, nose, and throat examination is otherwise normal. Chest examination
shows some left rib tenderness. Laboratory findings include an alcohol level
of 365 and a serum positive for cocaine. Complete blood count, electrolytes,
and coagulation studies are normal. X-rays show no fractures. The most
appropriate next step in management is to
A. discharge him to home from the emergency department
B. discharge him to a substance abuse rehabilitation center from the
emergency department
C. get a CT scan of the head
D. get an MRI of the spine
E. immediately start diazepam for alcohol withdrawal
Explanation:
The correct answer is C. This patient has had head trauma with loss of
consciousness. Although his neurologic examination is normal, a head CT
should still be done to look for a bleed. These are often subclinical on
admission and worsen over the first 1-2 days.
This patient sustained a head injury with loss of consciousness and requires
evaluation to determine if there is an intracranial bleed. He should not be
discharged home from the emergency department (choice A) without a more
thorough evaluation.
While substance abuse rehabilitation (choice B) may be necessary in the
future, right now he needs to be evaluated for an intracranial bleed and so
he should not be discharged at this time.
There is nothing in the history or physical examination to suggest a spinal
abnormality, so a spine MRI (choice D) is not indicated.
He should be given a benzodiazepine for alcohol withdrawal (choice E),
however this is not urgent and can wait until the acute issues have been
addressed.
A 39-year-old man is two days postoperative from a thyroid resection for papillary
carcinoma of the thyroid. Prior to his diagnosis, his medical history was unremarkable,
but 1 year prior to his surgery, he presented to his primary physician for evaluation of a
lump in his neck. This lump was eventually diagnosed as a cancerous thyroid nodule.
The patient was admitted to the hospital and his surgery went uneventfully. On visiting
him postoperatively, both you and the patient notice that his voice is hoarse. The most
likely reason for this is
18
A. damage to the recurrent laryngeal nerve
B. damage to the vocal cords
C. new tumor growth
D. residual tumor
E. upper respiratory infection
Explanation:
The correct answer is A. Knowledge of anatomy is critical in many areas of patient care.
This is especially true in surgery and in managing postsurgical complications. This
patient had work on the thyroid. The recurrent laryngeal nerve innervates all but one of
the muscles of the larynx and this nerve is adjacent to, if not directly opposed to the
thyroid gland. It is often damaged during thyroid resections and in fact, disclosure of
such possible damage is standard during the surgical consent process.
Although damage to the vocal cords (choice B) secondary to the endotracheal tube
could also account for the symptoms, such occurrences are rare and when they do
occur, happen as a result of prolonged intubation with an inappropriately large
endotracheal tube.
In 48 hours it is unlikely that there is new tumor growth (choice C).
Unless the original tumor was causing vocal cord paralysis by impinging upon the
recurrent laryngeal nerve, any residual tumor (choice D) should not either.
This patient has no evidence of any upper respiratory infection (choice E). Even if he
did however, given his proximity to surgery of the thyroid, the much more likely
explanation is that his hoarseness is secondary to his surgery or anesthesia, not a URI.
A 64-year-old woman with a diagnosis of chronic paranoid schizophrenia has moved
recently to town to live with her sister. She comes to the clinic for the first time, since she
needs to have a new doctor and needs her medication. She has been healthy most of
her life, and except for left hip replacement surgery, has had no other interventions or
treatments. She has been maintained well for years on thioridazine. She has not been in
the hospital for the past 15 years. She hears voices occasionally, but has developed
strategies to distract herself. She shares that she has not really had any other problems.
She goes to church, walks her sister's dog and watches TV. She denies any current
complaints. Her physical examination is unremarkable. Given this patient's history of
treatment and current age, the effect of thioridazine that she should be evaluated for is
A. cataracts
B. diarrhea
C. hypersalivation
D. hypertension
E. prolongation of the Q-T interval
Explanation:
The correct answer is E. Thioridazine belongs to the phenothiazine group of
19
antipsychotics that can cause prolongation of Q-T interval, inversion of the T wave, and
sometimes a bifid T or U wave. Changes are reversible. It has been noted that several
sudden deaths occurred secondary to cardiac arrests in patients treated with
thioridazine. The use of EKG periodically is thus advised.
Cataracts (choice A) have been described in beagle dogs treated with quetiapine.
Thioridazine can cause pigmentary retinopathy that is dose dependent. Because of its
anticholinergic effects, it may precipitate a glaucoma attack.
Diarrhea (choice B) is not a usual side effect of thioridazine. On the contrary, it has
anticholinergic properties, thus causing dry mouth, constipation, and paralytic ileus.
Hypersalivation (choice C) is a side effect of clozapine. Thioridazine, because of its
anticholinergic effects, causes dryness of the mouth and other mucous membranes.
Hypertension (choice D) is not a side effect of thioridazine. All phenothiazines tend to
cause hypotension, secondary to the action on alpha-adrenergic receptors.
A 74-year-old woman with diabetes and osteoarthritis is seen in your office
for a postoperative medical visit. She has been a long-time patient of yours
and five days ago underwent a left total knee replacement. Her hospital
course was uneventful and she comes to the office today for a follow-up on
her medical regimen. She continues to take her daily NPH insulin with good
control of her blood sugars. She also continues to take her oxycodone for
pain that was given to her in the hospital. She is involved in a physical
therapy rehabilitation program at the local hospital. The medication that is
most indicated for this patient at this time is
A. aspirin
B. enalapril
C. ibuprofen
D. subcutaneous unfractionated heparin
E. warfarin
Explanation:
The correct answer is E. This patient is post a total knee replacement and is
currently not on any anticoagulation therapy. The risk of deep venous
thrombosis and subsequent pulmonary embolism is very high in this
population and it is the standard of care to initiate warfarin or low molecular
weight heparin postoperatively for a period of 6 weeks to 6 months.
Oral aspirin (choice A) is an anti-platelet agent that has no role in the
prevention of DVT.
An ACE inhibitor (choice B) is, in the long-term, an excellent drug for this
patient given her diabetes. However, in the post-surgical period, the drug
that needs most consideration is for the most pressing issue.
A non-steroidal antiinflammatory agent (choice C) does not appear to be
indicated at this time as the patient appears to have reasonable pain control
with her opiate.
Subcutaneous unfractionated heparin (choice D) is used for the prevention
of DVT in immobile or hospitalized patients unable to ambulate. However,
the efficacy of post-orthopedic surgery, especially after joint procedures, is
20
very poor given the increased venous stasis.
A 61-year-old woman is status post a right total hip replacement 3 hours ago.
She underwent an uneventful replacement with hardware under spinal
anesthesia. She is brought the postanesthesia care unit (PACU) sedated but
alert and oriented to person, place, and time. Her past medical history is
significant only for hypertension and gout for which she takes allopurinol and
atenolol daily. On arrival to the PACU, she complains of some mild shortness
of breath and chest pain. Over the past 3 hours, her shortness of breath
significantly worsens and she has pleuritic chest pain on her right side. Her
temperature is 37.0 C (98.6 F), blood pressure is 100/60 mm Hg, pulse is
128/min, and respirations are 32/min. She appears markedly dyspneic, but is
alert and oriented to person, place, and time. Physical examination is
remarkable for clear lung fields and jugular venous pulse visible at 12cm with
the patient at 30 degrees elevation. There is no chest wall tenderness on
palpation. The most appropriate immediate action is to
A. administer a propranolol, intravenously
B. administer morphine for pain control
C. give the patient supplemental oxygen by face mask
D. order a chest radiograph
E. start warfarin therapy
Explanation:
The correct answer is C. If a transesophageal echo probe is placed in every
patient undergoing hip-fracture repair, the incidence of fat and particle debris
in the right atrium approaches 70%. In fact, a major risk of lower extremity
orthopedic procedures is pulmonary embolism due to fat or clot. The
intramedullary pressures generated during the repair are greater than 500psi
and are enough to cause venous extrusion of fat and other particulate matter
into the circulation. This patient almost certainly suffered a fat embolism and
her hypotension, elevated neck veins, tachycardia, and dyspnea reflect this
fact. She requires supplemental oxygen and possibly endotracheal
intubation since this syndrome will slowly progress over the ensuing 24
hours.
Giving the patient propranolol (choice A) for her tachycardia could be fatal .
Her tachycardia is a response to the low filling of the left ventricle secondary
to fat embolism present in her pulmonary arterioles and venules. Her blood
pressure is already tenuous despite her tachycardia and without it her
cardiac output and therefore blood pressure would likely drop precipitously.
There is no evidence that this patient has rate-related cardiac ischemia and
therefore is hypotensive.
Giving the patient morphine for pain control (choice B) is not appropriate.
The patient is not complaining of surgical site pain, but rather she is
manifesting pain from a pleuritic component of her embolism. Treating this
pain with morphine before the patient is stabilized would result in respiratory
collapse since the morphine would depress her respiratory rate and
21
therefore her oxygenation.
Ordering a chest radiograph (choice D) is appropriate to evaluate for any
pulmonary edema or obvious pulmonary infarction only after the patient has
received oxygen. Patients should be treated and stabilized prior to
diagnostic interventions.
Warfarin therapy (choice E) is incorrect in this case because this patient did
not suffer a pulmonary embolism due to clot, but from fat. Clot emboli are an
important source of morbidity and mortality in this population, but usually
beginning 24 hours post-procedure. This patient has had an acute
perioperative event that is too soon to be clot.
A 27-year-old man comes to the emergency department 30 minutes after
stepping on a nail in his garage. He was running late for his tennis game and
he was about to get into his car when he felt a sharp, dagger-like feeling in
his left foot. He looked down and saw a 3 inch nail protruding through the
sole of his tennis sneaker. He ran back into the house and had his wife drive
him to the hospital. He pulled the nail out of his foot in the car and says that
the nail appears intact. He says that he is generally very healthy and has not
been to the doctor in "ages". He cannot even remember his last "check-up",
but he assumes that it was when he was in his "late teens." He does not
know his immunization history but he recalls that he had all of the
recommended vaccines before going to college. Physical examination shows
a clean, stellate puncture wound on his left heel that appears to only
penetrate the superficial epidermis. There is no swelling or pain with the
movement of the toes and sensation is intact. He shows you the intact, shiny,
clean nail. After you irrigate, debride, and carefully inspect the wound, the
most appropriate next step is to
A. advise him to soak his foot in an iodine solution at home
B. give him a tetanus and diphtheria toxoid
C. order a CT scan of his left foot
D. request an immediate orthopedic surgery consult
E. send him home with a prescription for an antibiotic that covers
Pseudomonas aeruginosa
Explanation:
The correct answer is B. This patient has a puncture wound through a
sneaker, which is a very common presentation in the hospital and on the
boards. The major considerations in this case are immunization history and
penetration through the sneaker (risk of Pseudomonas aeruginosa
infection). Since he has not been to the doctor in a while, he probably needs
a tetanus and diphtheria toxoid at this time. This is recommended over just
the single antigen tetanus toxoid for persons >7 years old. The tetanus
immune globulin is an antitoxin that is usually only recommended for
contaminated, severe wounds, and when the patient is unsure if they have
ever received a tetanus vaccination or have an incomplete immunization
history. It should be given in this case because it is also recommended for
puncture wounds. Since this patient was up-to-date on his immunizations
22
before he went to college, approximately 10 years ago, he requires the Td
toxoid. The issue of a Pseudomonas aeruginosa infection is unsure at this
time, because he recently stepped on the nail, and there does not seem to
be any signs of infection (erythema, warmth, discharge, etc). Careful followup is mandatory if he is not started on an antibiotic.
Advising him to soak his foot in an iodine solution at home (choice A) is not
the most appropriate next step because this patient stepped on a nail and
requires a Td toxoid at this time. Also, some believe that using an iodine
solution does not prevent infection and may even delay healing.
A CT scan of his left foot (choice C) is unnecessary at this time. First of all, it
appears as if the nail is intact and there is no foreign material left in his foot.
But if you were to believe that some metal might be left in the wound, a plain
x-ray can be used to show metallic material. A CT scan is used for plastic,
wood, and other substances that are radiolucent.
It seems unnecessary to request an immediate orthopedic surgery consult
(choice D) for this simple, shallow, clean puncture wound on this patient's
heel. If the wound were deep and dirty or if were through the metatarsal
head (and the tennis sneaker), orthopedic surgery consultation should be
sought. However, this patient still requires a Td toxoid at this time, even if
you decide to call for the consult.
Sending him home with a prescription for an antibiotic that covers
Pseudomonas aeruginosa (choice E) is inappropriate at this time because
he requires a Td toxoid. The question as to whether or not you should
prescribe an antibiotic for this clean wound, after administering the toxoid is
a tough one. Technically, an antibiotic should be prescribed when signs of
infection are present, but since Pseudomonas osteomyelitis can be
devastating if it occurs, many people would probably prescribe an antibiotic
to try to prevent an infection. In either case, this answer is still incorrect
because you cannot send him home with an antibiotic without providing the
Td toxoid.
A 36-year-old man is brought to the emergency department after being
extricated from a motor vehicle crash. He is brought in by ambulance and it is
reported that he was a restrained passenger in a high-speed motor vehicle
accident. He was conscious at the scene but his legs were pinned under the
collapsed car. After being cut free, he was transported to the hospital. A rapid
assessment reveals that the patient has no drug allergies and had not drunk
alcohol prior to the crash. He is awake and alert with a Glasgow Coma score
of 15/15. His temperature is 37.0 C (98.6 F), blood pressure is 160/100 mm
Hg, pulse is 110/min, and respirations are 24/min. He denies pain in his neck
on palpation and has full range of motion. Physical examination shows clear
lungs, regular heart sounds, an open right humerus fracture, and bilateral
lower extremity injuries. His left leg is intact but swollen and erythematous.
The medical technicians report that the left leg was the pinned leg. His right
leg appears to have an open femur fracture. He has 2+ radial pulses
bilaterally. His left foot is cool compared with his right. The most ominous
physical finding would be
A. loss of deep tendon reflexes on the left
23
B. loss of dorsalis or posterior tibial pulses
C. pale color
D. paraesthesias to touch
E. tenderness on palpation
Explanation:
The correct answer is B. This patient likely has a compartment syndrome.
Because portions of the body such as the leg have fascial compartments
that are relatively noncompliant, tissue injury usually results in massive
increases in pressure. Once the pressure is great enough to overcome
arterial pulsations, ischemia and necrosis are imminent.
The loss of deep tendon reflexes on the left (choice A) is an early sign of
compartment syndrome and in conditions such as gluteal compartment
syndrome, is the most sensitive indicator of early compartment syndrome.
Since it is the most sensitive finding, it does not portend imminent loss of
tissue.
Pale color (choice C) and paraesthesias to touch (choice D) are also
findings in compartment syndrome but are not as ominous as loss of pulses.
Classically, of the six "Ps" of compartment syndrome: pallor, pain,
paresthesia, pulselessness, poikilothermia, and paralysis, these two are the
most nonspecific.
Because of the nature of the patient's injuries, tenderness on palpation
(choice E) is going to be present. Such a nonspecific finding is common to a
majority of injuries and is not necessarily representative of an injury severe
enough to cause a compartment syndrome.
A 53-year-old man who is status post renal transplant 3 years earlier, returns to the clinic
for routine follow up for multiple warts on his hands and feet. Since his last visit 1 month
ago he has been using large amounts of salicylic acid 40% plasters to treat his
widespread warts. He tells you in passing that over the past week he has developed
ringing in his ears. He currently takes many medications, including prednisone,
cyclosporine, and lansoprazole. His temperature is 38.3 C (101.0 F), blood pressure is
110/80 mm Hg, and pulse is 105/min. Physical examination shows approximately 20
verrucous papules on the plantar surface of the feet as well as involvement of multiple
fingers on the hands. The most appropriate next step in the management of this patient
is to
A. discontinue cyclosporine
B. discontinue lansoprazole
C. discontinue topical wart therapy
D. refer him for an audiology exam
E. tell him to continue current therapy until he sees an otolaryngologist about this
problem
Explanation:
24
The correct answer is C. Discontinuing topical wart therapy is the correct answer
because this patient is described to have widespread involvement of warts and covered
each lesion with 40% salicylic plasters. Long-term systemic prednisone causes atrophy
of skin, which results in increased absorption of topical salicylic acid. Salicylism
commonly present with tinnitus, vomiting, tachycardia, and fever. Severe poisoning can
result in seizures, coma, respiratory and cardiovascular failure, cerebral edema, and
renal failure.
Cyclosporine (choice A) and lansoprazole (choice B) are both incorrect because neither
medication typically result in tinnitus. In addition, there was no indication in the history
that this patient has overdosed on his medication.
Referral to audiology (choice D) or otolaryngology (choice E) are both incorrect at this
time. As mentioned above, salicylism can result in severe multiorgan failure if not
caught early on. The first and most important step to take in this patient is to stop
salicylic acid application. If tinnitus does not resolve, then a referral to a specialist is
necessary.
A 69-year-old man with osteoarthritis and diabetes is seen in your office for a
postoperative medical visit. He has been a long-time patient of yours and three days ago
underwent a left total knee replacement. His hospital course was uneventful and he
comes to the office today for a follow-up on his medical regimen. He continues to take
his daily NPH insulin with good control of his blood sugars. He also continues to take the
oxycodone for pain, that was given to him in the hospital. He is involved in a physical
therapy rehabilitation program at the local hospital. The most immediate life-threatening
risk to this patient in the perioperative period is
A. addiction to pain medication
B. a fall
C. a pulmonary embolism
D. renal failure
E. a venous thrombosis
Explanation:
The correct answer is C. Orthopedic surgical procedures, especially those involving the
hip and knee, have a very high incidence of intraoperative and postoperative deep
venous thrombosis. The greatest risk to come from the presence of a DVT is fatal
pulmonary embolism. Such events are, unfortunately, not rare.
Addiction to pain medication (choice A) is not a concern for this patient. There is a low
probability that a postsurgical patient, requiring opiate medications for pain from that
surgery, will become addicted to opiates. Belief in the contrary unfortunately is all too
common in medicine and many patients are denied needed pain relief because of such
irrational fears.
Falls (choice B) are always a concern with elderly patients, especially postoperatively.
However, in quantifying specific risks, the consequences associated with a fall are
much less than that of a pulmonary embolism.
Renal failure (choice D) is not likely at all in the near term. In the long term, it is very
likely that her diabetes will result in some form of renal failure.
25
Although venous thrombosis (choice E) is very common postsurgically, this entity poses
much less risk by itself, than a pulmonary embolism. A large number of these venous
thrombosis occur in the non deep venous system and therefore pose very little, if any,
risk to a patient.
A 45-year-old woman is admitted to the hospital after suffering an inhalational
burn injury. She was found in her living room by a fire-rescue team. She
presented to the hospital with laryngeal edema and was intubated in the
emergency department. The patient is now in the intensive care unit and is
mechanically ventilated and sedated. Her vital signs are stable; her body
temperature is now 37 C (98.6 F). After discussion with the team, it is decided
that the patient will need escharotomy and skin grafting procedures over the
ensuing 3 days. The decision is made to keep the patient sedated and
paralyzed. The drug that is contraindicated in this patient is
A. cis-atracurium
B. d-curare
C. diazepam
D. morphine
E. succinylcholine
Explanation:
The correct answer is E. Burn patients undergo a series of alterations in a
variety of tissues. In muscle, there is a proliferation of non-junctional
acetylcholine receptors. The consequence of this is that when a depolarizing
muscle relaxant such as succinylcholine is used in these patients, the serum
potassium may rise as much as 1.5 mEq/L. This may be enough to cause
cardiac arrest and death.
Cis-atracurium (choice A) and d-curare (choice B) are non-depolarizing
muscle relaxants that are safe in these patients. Owing to the increased
numbers of acetylcholine receptors, the dosing of this class of drugs needs
to be augmented.
Diazepam (choice C) is a benzodiazepine sedative that is also a mainstay of
therapy for burn patients.
Morphine (choice D) is a mu-agonist opiate and is a mainstay of pain control
in burn patients. Such patients typically require on average of 10mg per hour
as opposed to post-surgical patients that may receive 10mg every 12 hours
for pain control.
A 68-year-old woman is brought to the emergency department after being
rescued from her burning apartment building. She was found unconscious in
her bedroom, which was full of smoke and flames. On presentation, she has
extensive second and third degree burns over her legs and torso with an
estimated TBSA burn of 25%. The patient is stable with a blood pressure of
160/70 mm Hg and a pulse of 100/min. The body temperature is 36 C (96.8
F). The plan is to initiate fluid replacement. The most appropriate
management is to
26
A. infuse 1ml/kg x TBSA % burn of colloid solution on post burn day 2
for 24 hours
B. infuse 1ml/kg x TBSA % burn of colloid solution over 24 hours
C. infuse 4ml/kg x TBSA % burn of lactated ringers solution over 8
hours
D. infuse 4ml/kg x TBSA % burn of lactated ringers solution over 24
hours
E. infuse 4ml/kg x TBSA % burn of lactated ringers solution with one
half in the first 8 hours
Explanation:
The correct answer is E. Burn patients are acutely hypovolemic secondary
to massive capillary leak and tissue sequestration of fluid. Many formulas for
administering fluid resuscitation have been described, all in proportion to the
%TBSA burned. One half of the calculated resuscitation volume for the first
24 hours is given in the first 8 hours. The formula in this answer is the
Parkland formula, the most widely used resuscitation formula. Most centers
use crystalloid (normal saline or lactated ringers solution) for volume
resuscitation.
Infusing 1ml/kg x TBSA % burn of colloid solution on post burn day 2 for 24
hours (choice A) or 1ml/kg x TBSA % burn of colloid solution over 24 hours
(choice B) are colloid based resuscitation formulas. Most authors in the
Unites States believe the abundant evidence that outcome in burn patients
is not influenced by early colloid administration. If colloid has any role in the
management of acute thermal injury it is after the initial 24-hour post-burn
period.
Infusing 4ml/kg x TBSA % burn of lactated ringers solution over 8 hours
(choice C) and over 24 hours (choice D) are incorrect because of the
duration of infusions.
A 30-year-old man is brought to the emergency department following a motor vehicle
accident. He has no medical history, is on no medications, and has no allergies to any
medications. You are told that he was a restrained front passenger in a head-on
collision. He was observed to aspirate on the scene and was intubated by EMTs enroute for respiratory distress. He is intubated, sedated, and has a cervical collar in place.
His temperature is 37.2 C (99 F), blood pressure is 110/65 mm Hg, pulse is 110/min,
and respiratory rate is 23/min. His oxygenation is 100% on 100% oxygen. His has
decreased breath sounds over the right lower lung field and cardiac exam is significant
for regular tachycardia. A chest radiograph shows a right lower lobe consolidation. The
most appropriate next step in the management of his aspiration is
A. close monitoring with supportive care
B. administration of intravenous antibiotics
C. administration of intravenous steroids
D. administration of nebulized acetylcysteine
27
E. administration of nebulized beta agonists
Explanation:
The correct answer is A. Aspiration typically causes a 2-staged disease process. In the
immediate stage, it causes a chemical pneumonitis which can present with
consolidation on chest radiograph, fever, and/or leukocytosis. This pneumonitis can
resolve without medical intervention. Aspiration pneumonia typically presents as a
delayed complication and is typically seen in patients who are at a risk of having
nonsterile gastric contents (diabetic gastroparesis, small bowel obstruction, etc.). Not all
patients with a pneumonitis will go on to develop pneumonia. Therefore, in this patient it
is most appropriate to institute supportive care and monitor for the subsequent
development of pneumonia. This will decrease the risk of complications from
indiscriminate use of antibiotics such as selecting a resistant organism.
Administration of intravenous antibiotics (choice B) is not appropriate in this acute
setting since this patient most likely has an aspiration pneumonitis and not an aspiration
pneumonia as discussed above.
Administration of intravenous steroids (choice C) has not been shown to have any
clinical benefit in patients with aspiration, even in those with the aspiration pneumonitis.
Administration of nebulized acetylcysteine (choice D), a mucolytic agent, has not role in
the typical management of aspiration. This agent can be used in patients with
respiratory difficulty from thick mucous plugs, such as with cystic fibrosis.
Administration of nebulized beta agonists (choice E) has not been shown to have any
benefit following aspiration in patients who are not bronchospastic.
A 28-year-old woman comes to the office because of a 3-day history of a foul smelling
vaginal discharge. She has no other complaints. Pelvic examination shows a yellowishgreen frothy discharge, edema of the vulva, and petechiae on the cervix. A wet mount of
the discharge shows motile, flagellated organisms. You prescribe the appropriate
therapy in a single dose and she reminds you that she is breast-feeding her 3 month old
infant. At this time you should
A. advise her to discontinue breast-feeding indefinitely
B. prescribe a different pharmacologic agent
C. recommend that she pump and discard her milk for 24 hours and then resume
breast-feeding
D. take back the prescription and tell her that treatment of her infection is not
necessary
E. tell her that this medication does not appear in breast milk
Explanation:
The correct answer is C. This patient has an infection with Trichomonas vaginalis, and
should be treated with metronidazole. Metronidazole appears in breast milk, and the
effect on a breast-fed infant is unknown but may be of concern. Therefore, the
American Academy of Pediatrics recommends that the mother be given a single 2-g
dose of metronidazole and then that she pump and discard her milk for 24 hours. Then
she can resume breast feeding.
It is inappropriate to advise her to discontinue breast-feeding indefinitely (choice A). The
medication will not remain in her milk "indefinitely", and therefore the recommendation
28
is to discontinue breast-feeding for 12-24 hours after a single 2- gram dose of
metronidazole or to pump and discard her milk for 24 hours, and then resume breastfeeding.
Metronidazole is the treatment of choice for Trichomonas vaginalis. It is unnecessary to
prescribe a different pharmacologic agent (choice B) because all the patient needs to
do is discontinue breast-feeding for 12-24 hours after a single 2- gram dose of
metronidazole or to pump and discard her milk for 24 hours and then resume breastfeeding.
It is inappropriate to take back the prescription, and tell her that treatment of her
infection is not necessary (choice D), because this patient has a symptomatic
Trichomonas vaginalis infection that should be treated appropriately with metronidazole.
Since metronidazole does appear in breast milk, it is incorrect to tell her that it doesn't
(choice E).
A 72-year-old woman comes to the clinic because of a new onset of terrible headaches,
accompanied by pain in her left eye. She complains of seeing "halos" and is very
nauseated. Her vision is blurry. On the examination, her eye is hard and tender to touch
and the pupil is dilated and midfixed. She has no previous history of similar
ophthalmologic problems. Her previous history is significant for chronic lower back pain
after an injury in a motor vehicle accident years ago. She was recently started on new
medication by her neurologist to target insomnia because of the back pain and the pain
itself. To avoid further problems, the medication that should be discontinued, that is most
likely causing her current ophthalmologic problems is
A. amitriptyline
B. carbamazepine
C. gabapentin
D. propranolol
E. trazodone
Explanation:
The correct answer is A. Amitriptyline is a tricyclic antidepressant with the strongest
anticholinergic and sedative properties in its group. It is frequently used in low doses for
the treatment of chronic pain. Its strong anticholinergic effects can cause constipation,
urinary retention, dry mouth, acute glaucoma, paralytic ileus, etc.
Carbamazepine (choice B) is an anticonvulsant also used in the treatment of neuralgic
pain. It doesn't cause anticholinergic side effects, but it can have effects on liver
enzymes and bone marrow.
Gabapentin (choice C) is an anticonvulsant used as an adjunctive in the treatment of
pain. It has no anticholinergic side effects that would cause glaucoma.
Propranolol (choice D) is a beta blocker that is not used in the treatment of chronic pain.
It is actually used to treat a glaucoma attack, and thus should not be discontinued.
Trazodone (choice E) is an antidepressant with serotonergic properties. It is used for
the treatment of insomnia in low doses. Its main side effects include orthostatic
hypotension and sedation. It is not used in the treatment of pain.
29
A 45-year-old man is struck by a motor vehicle and is transported by a med flight to the
local emergency department. He is reported to be intoxicated with a Glasgow score of
12/15 on the scene of the accident. He was struck by a vehicle while crossing the street.
On arrival, the patient is awake and somewhat combative. He is alert to person only. His
temperature is 37.0 C (98.6 F), blood pressure is 150/80 mm Hg, pulse is 112/min, and
respirations are 20/min. Primary survey reveals a well-developed man in mild distress.
He is in a cervical collar. He has obvious lower extremity tibia fractures bilaterally and a
laceration on his forehead. The most appropriate manner to clear the patient's cervical
spine from injury is
A. cervical spine plain radiographs
B. CT scan of the neck
C. flexion, extension, and lateral rotation of the neck while assessing any pain
response from the patient
D. manual palpation of the patient's neck for any obvious injury
E. this patient cannot have his cervical spine cleared at this time
Explanation:
The correct answer is E. This patient is at high risk for a cervical spine injury and a key
component of any trauma evaluation is ensuring that none exists in situations such as
these. The two components of a successful evaluation for clearance are an imaging test
of some sort and a lucid, mentally clear patient. Once imaging is completed, the patient
is asked, during the examination part, to express pain or discomfort if there is any. This
patient is intoxicated and therefore is not able to reliably indicate pain during the
examination. Once he is sober the exam can be undertaken once his imaging study is
read as normal.
Cervical spine plain radiographs (choice A) or CT scan of the neck (choice B) are both
appropriate selections for imaging. Depending on the expertise of the radiologist, the
sensitivity of either for detecting cervical spine injury is equivalent. No imaging mode
however can be utilized in the absence of a physical examination.
Flexion, extension, and lateral rotation of the neck while assessing any pain response
from the patient (choice C) would be the appropriate sequence of events after a cleared
image for any obvious injury if he were not intoxicated.
Manual palpation of the patient's neck for any obvious injury (choice D) is not an
adequate physical examination. Exam must include lateral rotation and flexionextension of the patient's neck as well.
A 45-year-old man is struck by a motor vehicle and is transported by a med flight to the
local emergency department. He is reported to be intoxicated with a Glasgow score of
12/15 on the scene of the accident. He was struck by a vehicle while crossing the street.
On arrival, the patient is awake and somewhat combative. He is alert to person only. His
temperature is 37.0 C (98.6 F), blood pressure is 150/80 mm Hg, pulse is 112/min, and
respirations are 20/min. Primary survey reveals a well-developed man in mild distress.
He is in a cervical collar. He has obvious lower extremity tibia fractures bilaterally and a
laceration on his forehead. The most appropriate manner to clear the patient's cervical
spine from injury is
30
A. cervical spine plain radiographs
B. CT scan of the neck
C. flexion, extension, and lateral rotation of the neck while assessing any pain
response from the patient
D. manual palpation of the patient's neck for any obvious injury
E. this patient cannot have his cervical spine cleared at this time
Explanation:
The correct answer is E. This patient is at high risk for a cervical spine injury and a key
component of any trauma evaluation is ensuring that none exists in situations such as
these. The two components of a successful evaluation for clearance are an imaging test
of some sort and a lucid, mentally clear patient. Once imaging is completed, the patient
is asked, during the examination part, to express pain or discomfort if there is any. This
patient is intoxicated and therefore is not able to reliably indicate pain during the
examination. Once he is sober the exam can be undertaken once his imaging study is
read as normal.
Cervical spine plain radiographs (choice A) or CT scan of the neck (choice B) are both
appropriate selections for imaging. Depending on the expertise of the radiologist, the
sensitivity of either for detecting cervical spine injury is equivalent. No imaging mode
however can be utilized in the absence of a physical examination.
Flexion, extension, and lateral rotation of the neck while assessing any pain response
from the patient (choice C) would be the appropriate sequence of events after a cleared
image for any obvious injury if he were not intoxicated.
Manual palpation of the patient's neck for any obvious injury (choice D) is not an
adequate physical examination. Exam must include lateral rotation and flexionextension of the patient's neck as well.
A 40-year-old woman is admitted to the hospital because of fever, jaundice,
and abdominal pain. Laboratory studies on admission showed only an
elevated leukocyte count and an ultrasound of the abdomen showed dilated
bile ducts. An endoscopic retrograde cholangiopancreatogram (ERCP) was
performed and a stone was extracted. Today, she had 3 episodes of vomiting
and is complaining of "deep" epigastric pain that radiates to the back, is
worsened by lying flat on her back, and somewhat relieved by leaning
forward. Her skin is cool and clammy and her abdomen is distended and
tender. Laboratory studies show a leukocyte count of 21,000/mm3, an
amylase level of 450 U/L, and a lipase level of 400 U/L. She underwent a
laparoscopic appendectomy 3 years earlier. The most likely explanation for
the worsening of her symptoms is
A. erosion of the gastroduodenal artery
B. inflammation of the pancreas caused by endoscopic retrograde
cholangiopancreatography
C. intestinal obstruction caused by previous surgery
D. passage of a gallstone into the duodenum (gallstone ileus)
31
E. surreptitious alcohol consumption
Explanation:
The correct answer is B. This patient has developed acute pancreatitis
following the ERCP. This is a known complication of this procedure, which is
used to assess the cause, location, and extent of biliary obstruction and to
remove the stone. It is thought to be caused by injury to the ampulla of Vater
and/or retrograde injection of contrast material. Other complications of this
procedure include cholangitis, infected pancreatic pseudocyst formation, and
perforation.
Erosion of the gastroduodenal artery (choice A) is an unlikely cause of the
worsening of this patient's symptoms because this is typically associated
with a duodenal ulcer. The patient will often present with symptoms of a
duodenal ulcer (gnawing epigastric pain that is relieved by food) and then
the sudden worsening of symptoms and gastrointestinal bleeding. A
duodenal ulcer on the posterior wall may also penetrate through the
pancreas, liver, or biliary tree and may lead to increased pain and elevated
amylase levels. However, the patient in this case had a gallstone in a bile
duct, not a duodenal ulcer.
Intestinal obstruction caused by previous surgery (choice C) is an unlikely
cause for the worsening of her symptoms. Even though an intestinal
obstruction may be associated with elevated amylase levels, this patient's
prior appendectomy is less likely to be causing these symptoms than the
recent ERCP. Intestinal obstruction is characterized by crampy abdominal
pain, vomiting, obstipation, and abdominal distention. Also, the description of
the pain that it is worsened by lying down and somewhat relieved by leaning
forward is more consistent with pancreatitis than obstruction.
The passage of a gallstone into the duodenum (choice D) is associated with
a gallstone ileus and typically presents with symptoms of intestinal
obstruction (crampy abdominal pain, vomiting, obstipation, and distention). It
is usually associated with cholecystitis and fistula formation. This patient's
presentation is more consistent with pancreatitis than gallstone ileus.
While alcohol consumption (choice E) is one of the most common causes of
pancreatitis, it is very unlikely that this patient has been surreptitiously
consuming alcohol and there is no evidence presented in the case that
would suggest alcohol consumption. Therefore, it is more likely that her
pancreatitis is due to the ERCP than to alcohol.
A 47-year-old man comes to the office for a periodic health maintenance
examination. You have been treating him over the years for hypertension and
diabetes mellitus with enalapril and insulin. Lately you have been following
his cholesterol, which has been elevated despite an attempt at a low-fat diet,
a new moderate exercise program, and the cessation of smoking. In
discussing this issue with him, you decide that considering all of his other risk
factors for heart disease, it is time to start him on lovastatin. You explain the
risks and benefits associated with this drug, and you give him the prescription
before he leaves the office with instructions to call your office immediately if
he experiences any of the common side effects. Three weeks later, you
receive a frantic message from your answering service that this patient has
called to tell you that he has side effects from the medication. The complaint
32
that you expect to hear about when you return his call is
A. constipation
B. flushing
C. insomnia
D. muscle aches
E. nausea
Explanation:
The correct answer is D. Myositis and abnormal liver function are the two
main side effects associated with HMG-CoA reductase inhibitors, which are
one of the most common classes of drugs prescribed for elevated
cholesterol. While these side effects are relatively uncommon (1-2%), they
can be very serious and have led to fatalities. Myositis occurs more
frequently when combined with other cholesterol lowering agents.
Discontinuation of the medication is important if these symptoms occur.
Constipation (choice A) is associated with bile-acid resins, such as
cholestyramine.
Flushing (choice B) is associated with niacin, which is effective in decreasing
the synthesis of VLDL and LDL. Other side effects include tachycardia,
pruritus, nausea, diarrhea, elevated uric acid levels, and impaired glucose
tolerance.
Insomnia (choice C) is not commonly associated with any of the commonly
prescribed cholesterol-lowing agents. It is associated with methylphenidate,
which is prescribed for attention-deficit hyperactivity disorder.
Nausea (choice E) is associated with niacin, cholestyramine, and fibric acid
derivatives, such as gemfibrozil. It is not commonly associated with HMGCoA reductase inhibitors.
A 26-year-old man is admitted to the hospital after accidental ingestion of corrosive alkali
liquid. He denies any past medical or surgical history. The day before admission, he was
at a party with his friends where he consumed a lot of alcohol. At the end of the party, he
returned home and accidentally ingested corrosive alkali liquid from a bottle, mistaking it
for a bottle of water. At the time of admission to the hospital, he complained of
substernal chest pain and that he was feeling "really ill". Initial gastrointestinal contrast
study with water-soluble contrast did not reveal a gastrointestinal leak or perforation. His
vitals are stable with a tachycardia of 90/min. The next best step in management is to
A. continue to observe him in the hospital
B. order a CT scan of the chest
C. order an electrocardiogram
D. order an upper gastrointestinal study with barium
E. perform an upper gastrointestinal endoscopy
33
Explanation:
The correct answer is D. Chest pain, fever, tachycardia, subcutaneous emphysema,
dysphagia, and dyspnea are suggestive of an esophageal perforation. An esophageal
perforation may result from endoscopic procedures, external trauma, esophageal
disease, and spontaneous perforation from violent bouts of emesis. Prompt recognition
of an esophageal perforation is necessary to prevent delayed complications. Whenever
a perforation is suspected, a contrast study should be performed with water-soluble
contrast material. If this study does not demonstrate the perforation, it should be
repeated with barium. Barium is more accurate for a delineating esophageal leakage.
Contrast studies not only help in diagnosing esophageal rupture but also document the
level of injury, which has important implications for treatment.
Early diagnosis of esophageal perforation is necessary. Continued observation (choice
A) of a suspected esophageal perforation may lead to continued leakage and sepsis.
Results of esophageal perforation repair are better if the perforation is detected early.
A CT scan of the chest (choice B) is not sensitive enough as the first investigation in a
suspected esophageal perforation. The contrast leak from perforation may show up on
the CT scan. Pleural effusions may provide indirect evidence of esophageal perforation.
An electrocardiogram (choice C) is not indicated in a young patient with a suspected
esophageal perforation.
An upper gastrointestinal endoscopy (choice E) is contraindicated as the first
investigational choice in a suspected perforated esophagus. Rigid esophagoscopy may
be indicated to extract the foreign body from esophagus.
An unconscious 21-year-old man is rushed into your emergency department following a
high speed motor vehicle accident. He has bilateral upper extremity fractures and no
obvious lower extremity fractures. While your team members manage his airway and
stabilize his neck, you are given the responsibility of obtaining vascular access. His
blood pressure is 105/54 mm Hg and pulse is 118 /min. Your line should be placed
A. above the inguinal ligament where the femoral vein crosses the femoral artery
B. in the femoral artery lying lateral to the femoral vein
C. in the femoral artery lying medial to the femoral vein
D. in the femoral vein lying lateral to the femoral artery
E. in the femoral vein lying lateral to the femoral nerve
F. in the femoral vein lying medial to the femoral artery
G. in the internal jugular vein
Explanation:
The correct choice is F. This question tests your knowledge of two topics, management
of the trauma patient and knowledge of the anatomy of the femoral canal. Knowledge of
the anatomy of the femoral canal will allow for appropriate placement of a large bore
catheter into the venous system. From lateral to medial, the anatomy of the femoral
canal is femoral nerve, femoral artery, the femoral vein, and lymphatic channels. The
only answer that meets all of the necessary criteria is in the femoral vein lying medial to
34
the femoral artery. An unstable trauma patient requires venous access with a large bore
catheter that has multiple ports to allow for the administration of fluids, blood products,
and medications simultaneously. In this patient the catheter should be placed in the
groin.
Above the inguinal ligament (choice A) the femoral artery and vein are named the
external iliac artery and vein.
Arterial access (choices B and C) will not allow for the delivery of medications, fluids,
and blood products directly to the heart for systemic distribution.
In the femoral vein lying lateral to the femoral artery (choice D) is incorrect because as
stated above, the femoral vein is medial to the femoral artery.
In the femoral vein lying lateral to the femoral nerve (choice E) is incorrect because as
stated above, the femoral vein is medial to the femoral artery.
The patient's jugular vein (choice G) will be difficult to access as his neck requires
stabilization in a collar until appropriate radiographic studies are obtained. Upper
extremity access is limited secondary to bilateral fractures.
A 72-year-old man returns to the office following an esophagoscopy done
earlier today because of a progressive dysphagia for the past 6 months. A
0.8-cm ulcerated lesion is found and a biopsy is taken. He is now, 4 hours
later, complaining of severe chest pain. His temperature is 38.3 C (101.0 F),
blood pressure is 130/80 mm Hg, pulse is 125/min, and respirations are
28/min. Physical examination shows crepitation in the neck and a crunching
sound over the heart. You call an ambulance and escort him to the hospital.
After you obtain a surgical consult, the most appropriate next step in
management is to
A. obtain an immediate bronchogram
B. order an immediate esophagogram with a water-soluble agent
C. perform an immediate bronchoscopy
D. perform an immediate thoracotomy
E. repeat the esophagoscopy to evaluate the biopsy site
Explanation:
The correct answer is B. This patient most likely has an esophageal
perforation, which is a serious, life-threatening complication of
esophagoscopy. The signs and symptoms may include chest pain near the
rupture site, fever, crepitus, and air in the mediastinum behind the heart. An
immediate esophagogram with a water-soluble agent is indicated. The
treatment is wide drainage by thoracotomy.
A bronchogram (choice A) is used to define the bronchial anatomy, and is
used to evaluate a patient's bronchiectasis. It is not indicated in a case of
esophageal perforation.
Bronchoscopy (choice C) is used to evaluate a patient with a suspected
bronchial obstruction or endobronchial disease. It is not indicated in this
patient with a suspected esophageal perforation.
Thoracotomy (choice D) is the treatment for an esophageal perforation,
35
however an immediate esophagogram with a water-soluble agent should be
performed to establish the diagnosis.
Repeating the esophagoscopy to evaluate the biopsy site (choice E) is not
the preferred method to identify an esophageal perforation. An
esophagogram is better to evaluate for a suspected perforation.
A 15-year-old boy is brought into your office by his parents because of numbness and
tingling in his feet, slowly increasing over the last several weeks. He has attention deficit
hyperactivity disorder for which he has been taking methylphenidate for the past 4 years.
In addition, he had a positive PPD skin test after screening upon entrance to high school
and was placed on isoniazid 6 months ago. He takes no other medications and has no
allergies. His parents feel that he has become more difficult to control since entering
high school and the father confides to you that he suspects these "latest complaints" are
a method of avoiding physical education class. He answers your questions readily and
reports that gym is actually his favorite class at school. He denies any problems at
home, with friends, or at school. On physical examination, you find nothing abnormal
except hyperesthesia over a stocking distribution of both lower extremities. Deep tendon
reflexes, motor function, and sensation are intact. This patient's current condition would
have most likely been prevented by supplementation with
A. folate
B. pyridoxine
C. riboflavin
D. thiamine
E. vitamin E
Explanation:
The correct answer is B. One of the side effects of isoniazid therapy is peripheral
neuropathy, which occurs more frequently in children. Supplementation with pyridoxine,
25-50 mg PO qd, is commonly used to decrease the incidence of neuropathy.
Pyridoxine is also known as vitamin B6.
Folate (choice A), riboflavin (choice C), thiamine (choice D), and vitamin E (choice E),
are not helpful in preventing this adverse effect.
Seven days ago, a 62-year-old man underwent a colonic resection for carcinoma. His
postoperative stay is prolonged because of pneumonia. His past history is significant for
insulin-dependent diabetes mellitus, hypertension, congestive heart failure, and
bronchitis. He has a 30-pack year smoking history. Colonic resection was uneventful. He
is tolerating a regular diet and is scheduled to be discharged home the next day. You are
called by the nurse to check his wound as she noticed a new serous discharge. His
temperature is 37.0 C (98.6 F), pulse is 80/min, blood pressure is 130/80 mm Hg, and
blood glucose by finger stick is 240 mg/dL. Abdomen is soft and non-tender. There is no
induration, erythema, or crepitus of the wound. Copious amounts of serous brownish
liquid is expressed from the wound on gentle pressure. Fluid is expressed
spontaneously on coughing from both upper and lower parts of the wound. The most
36
appropriate next step is to
A. control his blood sugar
B. order diuretics
C. order dry dressings to be continued
D. prescribe a 1-week course of antibiotics
E. prepare the patient for another laparotomy
Explanation:
The correct answer is E. Abdominal wound dehiscence due to poor facial healing
occurs typically between 7-10 days after laparotomy. Patients with chronic bronchitis,
diabetes, and a smoking history, are prone to poor facial healing. Poor wound closure
technique is the main cause of wound dehiscence. New onset serous discharge from a
laparotomy wound should raise the suspicion of wound dehiscence. Facial integrity
should be checked when dehiscence is suspected. The pouring of fluid from both upper
and lower parts of the wound should lead you to suspect complete dehiscence, which
needs to be fixed by a re-laparotomy.
Control of blood sugar is essential in a diabetic for better wound healing, but
dehiscence of the wound requires re-laparotomy and suturing of the wound, not just
controlling the blood sugar (choice A).
Congestive heart failure causes edema of the lower extremities and pulmonary
congestion. Serous discharge from the wound is not a sign of congestive heart failure.
Diuretics are not helpful in the healing of the wound (choice B).
Simple application of dressings to the dehisced wound would not prevent complete
dehiscence and evisceration. Dry dressings (choice C) are not an option in treating
complete wound dehiscence.
Antibiotics are indicated in wound infection, but not in wound dehiscence (choice D).
A 47-year-old man is brought into the emergency department by ambulance
after a motor vehicle accident. He is conscious, alert and, besides the
lacerations on his scalp, he seems to be otherwise well. His neck is stabilized
by a brace and he asks you to remove it. You assess his neurologic exam to
be non-focal, but you obtain cervical neck films anyway. No cervical spine
abnormalities are noted and you remove the cervical collar. Suddenly, the
patient's pulse rises to 130/min, his blood pressure drops to 80/50 mm Hg,
and his respiratory rate increases to 35/min. His physical examination reveals
jugular venous distension, peripheral and perioral cyanosis, absent breath
sounds on the left side of his chest, a tachycardic but otherwise normal
cardiac examination, and a tender abdomen in the left upper quadrant. The
most appropriate initial step is to
A. insert a needle thoracostomy into the second intercostal space
B. insert a tube thoracostomy into the pleural space
C. intubate the patient for hypoxemic respiratory failure
37
D. order a portable chest x-ray
E. perform a diagnostic peritoneal lavage for a splenic rupture
Explanation:
The correct answer is A. This patient presented with the classic signs and
symptoms of a tension pneumothorax, which presents a unilateral loss of
breath sounds with hypertympany, shift of the trachea away from the injured
side, and jugular venous distension. The diagnosis is based upon a
characteristic history and examination. He requires a needle thoracostomy
into the second intercostal space.
The patient may require a tube thoracostomy (choice B) for his tension
pneumothorax; however a needle thoracostomy to decompress his lung
would be the first step prior to insertion of a chest tube.
Intubating this patient (choice C) may be required if his hypoxemia doesn't
resolve with a needle thoracostomy and supplemental oxygen. However, it
would be premature as an initial first step
Instead of a diagnostic test such as a chest x-ray (choice D), a tension
pneumothorax needs to be emergently treated. Decompression is
accomplished by inserting a large-bore needle into the second intercostal
space in the midclavicular line on the affected side.
A diagnostic peritoneal lavage (choice E) may be required to evaluate the
patient's abdominal tenderness. His hypotension and tachycardia could be
secondary to acute blood loss, especially from an intraabdominal source like
a splenic rupture. However, airway and breathing must be evaluated and
treated prior to circulation.
A 24-year-old third-year medical student is brought to the emergency department from
the operating room after she began to have trouble breathing after she put on her gown,
latex gloves, and mask. This was going to be her first time "scrubbing in" to a case on
the first day of her surgery rotation. Another medical student who was "scrubbed in" to
the case, tells you that the she was grasping at her throat and attempting to rip off her
gown. She is now on a stretcher, gasping for air. She is brought into a room and all of
her clothes are taken off, revealing red hands and an eruption of well-circumscribed,
erythematous, raised lesions covering her entire body. Her blood pressure is 70/50 mm
Hg, pulse is 110/min, and respirations are 45/min. She has marked laryngeal edema and
audible wheezes. The other student says that this is the second day of their third year,
and that the only other patient contact that they have had was during the first year when
they went as a group to evaluate a patient in respiratory isolation. At this time the most
correct statement about her condition is:
A. Her rash is unrelated to her respiratory symptoms
B. Her symptoms are caused by the anxiety and stress from the first surgical
experience
C. Her symptoms are due to an infectious pathogen
D. Her symptoms are the result of bioactive mediators released when exposed to
an antigen
38
E. She should be given indomethacin immediately to reduce the respiratory
symptoms
Explanation:
The correct answer is D. This patient is experiencing an anaphylactic reaction, which is
a life-threatening response in a sensitized individual to a specific antigen, most likely
the latex in the gloves, and is associated with the release of lipid mediators, secretory
granule preformed mediators, and cytokines. The manifestations typically occur within
seconds to minutes of exposure and include respiratory distress, vascular collapse,
urticaria and angioedema, gastrointestinal symptoms, and shock. The reaction involves
IgE-dependent activation of mast cells, basophils and the release of mediators such as
histamine, cytokines, and lipid mediators. The immediate treatment involves the ABCs
(intubation, oxygen, and intravenous saline), epinephrine, a vasopressor agent such as
dopamine, an antihistamine such as diphenhydramine, and glucocorticoids to alleviate
later recurrence of symptoms.
It is incorrect to say that the rash is unrelated to her respiratory symptoms (choice A)
because they are both associated anaphylactic reactions. Anxiety and stress from the
first surgical experience (choice B) may be associated with vasovagal (vasodepressor)
syncope, which is the cause of the common "faint". It frequently occurs during periods
of stress or fear and often occurs in medical students during new experiences. The
symptoms include hypotension, bradycardia, nausea, pallor, diaphoresis, and a sudden,
transient loss of consciousness. Lying down and elevating the legs reverses the
symptoms. This is very different from a life-threatening anaphylactic reaction where
hypotension, tachycardia, respiratory failure, shock, and skin manifestations occur.
Assuming a supine position during an anaphylactic reaction does not reverse the
symptoms.
This patient is most likely experiencing an anaphylactic reaction from the latex gloves,
not an infectious pathogen (choice C). Pathogens may cause septic shock by releasing
toxins that then activate cytokines, platelet activating factors, arachidonic acid
metabolites, and humoral defense systems. This manifests as hypotension,
tachycardia, changes in mental status, and cold extremities. The laryngeal edema and
specific rash that this patient has are more consistent with an anaphylactic reaction
than septic shock.
She should definitely not be given indomethacin immediately to reduce the respiratory
symptoms (choice E) because it is a nonsteroidal anti-inflammatory drug that may
cause similar life-threatening respiratory reactions in patients with asthma. You do not
know her medical history. Also, this is not part of the management of anaphylaxis,
which includes epinephrine, saline, vasopressors, and intubation.
A 15-year-old boy is brought to the office by his father for a physical examination before
summer camp. Since baseball season is in full swing, and he is the pitcher on the varsity
team, he comes to the office in uniform to be able to go straight to the game after the
appointment. He talks incessantly about the game and his father chimes in every so
often to show his support for his son. During the examination, the father talks non-stop
about getting his son a full baseball scholarship to an ivy league college. He has been in
touch with all of the coaches at the schools. You are surprised to see how muscular the
boy has become since his physical examination the previous year. You inquire about
weight-lifting, and he says that he works-out in their basement daily. His father bought
him a full weight set for his birthday. You suspect that he is taking anabolic steroids.
39
During the evaluation, particular attention should be given to
A. funduscopic examination
B. neurologic reflexes
C. respiratory examination
D. testicular size
E. thyroid examination
Explanation:
The correct answer is D. Testicular atrophy may be seen with anabolic steroid usage,
which is common in athletes trying to increase muscle mass and strength, and maintain
a "winning edge". Athletes may use anabolic steroids in very high doses that lead to
many adverse effects that may include liver disease, gynecomastia, impotence, acne,
striae, edema, increased cholesterol, aggression, and depression.
Anabolic steroids are not typically associated with abnormalities in the funduscopic
examination (choice A), neurologic reflexes (choice B), respiratory examination (choice
C), and thyroid examination (choice E).
A 78-year-old woman with non-insulin dependent diabetes mellitus is
admitted to the hospital because of vomiting, abdominal pain, and
obstipation. Her past surgical history is significant for appendectomy,
hysterectomy, and a cholecystectomy. She was found at home very
dehydrated and vomiting. On admission she is lethargic. Her blood pressure
is 100/60 mm Hg and pulse is 100/min. Her mucous membranes are dry, her
abdomen is distended, but there are no peritoneal signs. An incarcerated
hernia is found and she undergoes a laparotomy. At laparotomy, a bowel
resection is performed for ischemic bowel segment and the hernia is
repaired. She received 100 cc of intravenous fluids per hour postoperatively.
On the third postoperative day, she spiked a fever of 39.1 C (102.4 F).
Physical examination reveals dry oral mucous membrane and a tender, 3 cm
nodule in front of the right ear. The most appropriate next step in the
management is to
A. begin aggressive hydration
B. begin appropriate oral hygiene
C. prepare her for immediate surgical drainage
D. send blood cultures and await sensitivities
E. start empiric therapy for Staphylococcus aureus
Explanation:
The correct answer is E. Acute suppurative parotitis is a severe, life
threatening infection of the parotid gland, most often seen in elderly or
debilitated patients who are severely dehydrated. Pathogenesis is thought to
be related to stasis within salivary ducts as a result of increased viscosity.
40
Staphylococcus aureus is the organism most often found in this severe
infection. Initial treatment with appropriate intravenous hydration,
sialogogues, and antibiotics against Staphylococcus may be successful. If
improvement is not seen within 12 hours of initiating this treatment, surgical
drainage is warranted.
Aggressive hydration (choice A) is essential to correct the dehydration in this
elderly patient, although treatment of acute suppurative parotitis should be
started at the time.
Oral hygiene (choice B) is not the main therapy for acute suppurative
parotitis.
Immediate surgical drainage (choice C) is not indicated unless the patient
does not improve in 12 hours.
Blood cultures (choice D) are essential to isolate the causative organism, but
empiric antibiotic therapy should not be delayed awaiting the sensitivities.
A 6-year-old girl is brought to the emergency department 10 hours after being bitten on
her face while she was playing "dog" with her friend. The two girls were on the ground
with the family dog, laughing, growling, and "biting the air," when all of a sudden the
patient felt a "sharp, stabbing pain" on her right check. She ran to the mirror and started
screaming when she saw blood on her face. "It all happened so fast" that nobody is
exactly sure what happened. Both the friend and the dog had blood on their mouths, but
they are unsure if it was the girl or the dog who actually broke the patient's skin. The dog
and both girls are up-to-date with all vaccinations. Physical examination shows a 2-cm
laceration on her right check. The surrounding tissue is tender. You request a plastic
surgery consult after culturing and copiously irrigating the wound. The most appropriate
additional treatment is to
A. administer ampicillin/sulbactam, intravenously
B. administer metronidazole, intravenously
C. give her clindamycin, orally
D. give her penicillin V, orally
E. provide no antibiotic therapy at this time
Explanation:
The correct answer is A. This patient has a bite wound on her face that was either
caused by a dog or another human, and she requires antibiotic prophylaxis. The most
appropriate of all of the choices is ampicillin/sulbactam, intravenously because it covers
the most likely pathogens such as S. aureus, P. multocida, H. influenzae, and Blactamase-positive oral anaerobes. This girl requires treatment because all human bites
require antibiotics, and dog bites to the face typically require treatment.
Metronidazole (choice B) is not the preferred antibiotic prophylaxis for human or dog
bites. It is given for anaerobic infections, giardiasis, trichomonas, pseudomembranous
colitis, and bacterial vaginosis.
Clindamycin (choice C) may be given to cover Gram-positive cocci and anaerobes.
However, it is best given along with trimethoprim-sulfamethoxazole to cover the other
41
potential pathogens. However, this is not the preferred antibiotic regimen. Since there is
a chance that she will be admitted to the hospital and because it is a facial wound that
might have been inflicted by another person, an intravenous route of administration is
best (even if it is one dose in the emergency department and then the patient is
discharged with a prescription for oral therapy).
Penicillin V, orally (choice D) is not the preferred antibiotic prophylaxis for human or dog
bites because of poor coverage of some anaerobes and Gram-negative organisms, and
most of the Gram-positive organisms present in human or dog bite infections. Penicillin
V is typically used to treat streptococcal infections.
No antibiotic therapy is indicated (choice E) in non-extensive dog bites that do not
appear infected, do not occur on the face or involve a bone or joint, and when there are
no co-morbidities. Human bites always require antibiotics and often require hospital
admission. Since they are unsure as to the source of the bite, and it involved the face,
antibiotics are indicated.
A 24-year-old man returns to your office 10 days after a right knee
arthroscopy for a medial meniscus tear. The patient originally injured his knee
playing football with his brothers and an MRI disclosed a meniscal tear. The
patient has no other medical history and underwent a successful arthroscopic
meniscus removal under general anesthesia. He reports that since his
surgery his voice has been hoarse and he often feels liquid get into his
"windpipe". He should be told that:
A. He likely has an upper respiratory tract infection
B. He should have his vocal cords evaluated by a otolaryngologist
C. His symptoms are caused by a thyroid problem
D. His vocal cords were damaged from the endotracheal tube
E. It is normal to have a sore throat and hoarseness after surgery
Explanation:
The correct answer is B. One of the risks of general anesthesia is damage to
the vocal cords by direct trauma. Although rare, it does occur and it
manifests itself in just the way this patient presents; hoarseness after
surgery that does not improve after one week. Any such patient should have
a full ENT evaluation to determine if there is any cord paralysis or paresis.
This patient has no evidence of any upper respiratory infection (choice A).
Even if he did however, given his proximity to general anesthesia and an
endotracheal tube, it is much more likely that his symptoms are due to this
event and not a URI.
Given that the patient has recently had a general anesthetic, the likelihood
that there is a thyroid problem (choice C) responsible for his hoarseness is
very small.
Although it is possible that his vocal cords were damaged from the
endotracheal tube (choice D), this has yet to be established by physical
examination of the cords. Although likely, making the definitive diagnosis
and failing to visualize the cords could result in a missed diagnosis of
42
laryngeal polyp or other disturbance causing the hoarseness.
Although it is normal to have a sore throat and hoarseness after surgery
(choice E), this is only true in the immediate postoperative period of less
than 24 hours. There is nothing normal about residual hoarseness at 10
days postoperatively.
The mother of a 3-year-old patient of yours, calls the office after the child
banged his head against the coffee table in their family room. The mother is
absolutely frantic, saying that her son has been "screaming and crying" for
the past 45 minutes and that she cannot calm him down. She tells you that he
did not lose consciousness. You advise her to bring him into the office, even
though you just finished seeing your last patient and were just planning on
leaving for the evening. When they arrive, you notice that the boy's eyes
appear very red from crying, but that he has calmed down. The mother is still
very concerned. They have both been patients of yours for many years, and
they have always been very healthy and compliant. You notice that there is a
0.5-cm edematous area on the back of his head. The skin is intact over the
wound. The remainder of the physical examination, including a complete
neurologic and funduscopic examination, is unremarkable. After calming the
mother down, the most appropriate next step is to
A. advise them to go to the emergency department for observation
B. obtain a skull radiograph
C. order a CT scan of the head
D. recommend regular monitoring and observation for any
abnormalities, and if they arise, that they should go to the emergency
department
E. report the case to the child protective service agency
Explanation:
The correct answer is D. This boy has a minor closed head injury with no
loss of consciousness, which is one of the most common injuries in children.
All children "bump" their heads at some point. It is important to do a physical
examination, including a complete neurologic and funduscopic examination,
and if this is normal, be able to recognize that further studies are generally
not indicated. Regular monitoring and observation for any abnormalities can
be done by a competent caregiver. If this occurred during the day, you may
consider having them stay in your waiting room for a little while, but since
you are leaving for the night and the mother has always been responsible
and compliant, you can send them home for observation.
It is inappropriate to advise them to go to the emergency department for
observation (choice A). This child had a minor head injury with no loss of
consciousness and he has a normal neurologic and funduscopic
examination. It is very unlikely that he has an intracranial injury, and
therefore, as long as you know that the mother is a competent caretaker,
you should send them home after advising her to seek assistance if he
begins to deteriorate.
43
It is unnecessary to obtain a skull radiograph (choice B) at this time in this
child with a minor closed head injury without a loss of consciousness. He
does not have any signs of a skull fracture, which include battle signs
(ecchymoses behind the ear), a palpable depression, or blood in the ear and
therefore it is very unlikely that the radiograph will show a skull fracture.
Also, even if he did have a skull fracture that does not necessarily mean that
he has an intracranial injury.
A CT scan (choice C) is not indicated at this time in this patient with a minor
closed head injury, no loss of consciousness, and a normal neurologic and
funduscopic examination. Studies have shown that the risk of intracranial
injury is negligible in this situation, and that fewer than 1 in 5,000 patients
with minor closed head injuries and no loss of consciousness have
intracranial injuries that require medical or neurosurgical intervention.
Since this case states that they are very healthy and compliant patients and
the physical exam does not reveal any abnormalities besides this head
wound that seems consistent with the story the mother told you, it is
inappropriate to report the case to the child protective service agency
(choice E). It is always important to be aware of signs of child abuse, such
as many emergency room visits, many wounds in various stages of healing,
implausible and inconsistent stories, and bringing the child in a while after
the injury took place. This case does not seem to fit this description.
A 31-year-old man is admitted to the hospital after a motorcycle accident. He
was fully alert and oriented upon arrival and a primary survey revealed a left
pneumothorax and small hemothorax with multiple rib fractures bilaterally.
Other than some abrasions and lacerations requiring sutures, the patient was
otherwise unremarkable. Initial laboratory results and electrocardiograms
were unremarkable. After a chest tube was placed, the patient had reinflation
of his left lung as demonstrated by chest radiograph. He is now concerned
about further complications. The complication that this patient is at greatest
risk for is
A. aortic dissection
B. empyema
C. myocardial infarction
D. pneumonia
E. subarachnoid hemorrhage
Explanation:
The correct answer is D. Rib fractures are exquisitely painful. The magnitude
of the pain forces people to halt their inspiratory efforts after small tidal
volumes and prohibits them from effectively coughing and clearing
secretions. This pattern of rapid, shallow breathing with no coughing
predisposes patients to profound atelectasis of the lungs and infection due
to residual secretions. Therefore he is at greatest risk for developing
pneumonia. Often, these patients will receive rib blocks or thoracic epidural
44
catheters for pain management. Opiates are not very effective since they
produce somnolence, often worsening the respiratory problems.
All major chest trauma carries the possibility of aortic dissection (choice A),
which is why chest radiographs are obtained to rule out a widened
mediastinum. Although the negative predictive value of such films is not
100%, the fact that this patient has no chest or back pain coupled with a
normal chest radiograph makes him especially low risk for this complication.
An empyema (choice B) is an infection of the pleural space. Since this
patient has had his hemothorax drained, his risk of subsequent infection is
essentially the same as the general population. His chest tube confers only
a minimal increased risk of empyema.
With major myocardial contusions from blunt chest trauma, both direct
myocardial injury as well as coronary dissection can result in infarction
(choice C). This patient has a normal EKG, no symptoms of chest pain, and
no evidence of major blunt chest trauma in the form of a hematoma or bruise
across the sternum. For these reasons, his risk of infarction is quite low.
The most common cause of subarachnoid hemorrhage (choice E) is trauma.
The presentation of SAH however is often directly after the trauma, and it is
most often head trauma as the precipitating event. It is very rare for a SAH
to have a delayed manifestation of hours or days after the trauma was
suffered.
In early July, a 23-year-old female marathon runner is brought to the emergency
department after collapsing during a 10 mile run. Her past medical history is significant
for hypothyroidism and she is currently being treated with thyroid replacement
hormones. On presentation she appears somnolent. Her skin is clammy and she is
profusely diaphoretic. Her pulse is 110/min and blood pressure is 85/50 mm Hg. Her
respirations are slow and shallow. Pulse oximetry measures 95% oxygen saturation on
room air. A rectal thermometer shows a core body temperature of 40.8 C (105.4 F). Her
lung sounds are clear. No murmurs were appreciated on cardiac auscultation. A
neurologic examination shows dilated but reactive pupils and intact reflexes. There are
noticeable petechiae over her trunk and right arm. Laboratory studies show:
A chest x-ray shows clear lungs and a normal cardiac silhouette. An electrocardiogram
shows sinus tachycardia. Immediate cooling measures and an intravenous saline bolus
are initiated. The patient is admitted to the intensive care unit for further management.
Additional laboratory tests that are helpful in management of this condition are
A. alanine aminotransferase, aspartate aminotransferase, alkaline phosphatase
45
B. creatine kinase, urinalysis, fibrin split products
C. lactate dehydrogenase, cardiac troponin, C-reactive protein
D. serum albumin, erythrocyte sedimentation rate, anti-nuclear antibody
E. thyroid stimulating hormone, thyroglobulin binding index, free thyroxine level
Explanation:
The correct answer is B. This patient is an athlete who presented to the emergency
department with exertional heat stroke. Heat stroke can occur in athletes who exert
themselves in conditions of high humidity and temperatures. Clinical signs of this
condition include a core body temperature of >40.5 C (or 105 F), profuse sweating, and
mental status changes. Patients with exertional heat stroke are at risk for disseminated
intravascular coagulation and rhabdomyolysis. This patient's presentation, prolonged
coagulation times, evidence of petechiae, and low platelet count, is consistent with
disseminated intravascular coagulation, and a positive test for fibrin split products will
confirm the diagnosis. Secondly, heat stroke involves direct thermal damage to muscle
tissue resulting in rhabdomyolysis. Muscle myoglobin deposition in renal tubules results
in acute tubular necrosis and renal failure. A creatine kinase assay of >10, 000 UI/L will
show skeletal muscle breakdown significant enough to cause acute renal failure. In
addition, a urinalysis can detect the presence of urine myoglobin and monitor urine pH.
Treatment for rhabdomyolysis involves intravenous saline infusions to maintain a high
urine output and alkalization of urine with a target pH of 7-8 to prevent myoglobin
deposition.
Alanine aminotransferase, aspartate aminotransferase, alkaline phosphatase (choice A)
are predominantly liver function tests. Although they may be abnormal in this patient,
they are not useful in the management of heat stroke.
Lactate dehydrogenase, cardiac troponin, and C-reactive protein (choice C) are useful
in the diagnosis of myocardial infarction and do not aid in the therapy of heat stroke or
rhabdomyolysis.
Serum albumin, erythrocyte sedimentation rate, and an anti-nuclear antibody test
(choice D) are useful in evaluating a rheumatological process. They do not add
information pertinent to the management of this patient.
The patient does have hypothyroidism, and these tests (choice E) would aid in
treatment of her hypothyroidism. However, they are not helpful in treating her current
condition.
46